Sie sind auf Seite 1von 69

G.R. No.

76464 February 29, 1988

TESTATE ESTATE OF THE LATE ADRIANA MALOTO, ALDINA MALOTO CASIANO,


CONSTANCIO MALOTO, PURIFICACION MIRAFLOR, ROMAN CATHOLIC CHURCH OF MOLO,
AND ASILO DE MOLO, petitioners,
vs.
COURT OF APPEALS, PANFILO MALOTO AND FELINO MALOTO, respondents.

SARMIENTO, J.:

This is not the first time that the parties to this case come to us. In fact, two other cases directly
related to the present one and involving the same parties had already been decided by us in the
past. In G.R. No. L-30479, 1which was a petition for certiorari and mandamus instituted by the petitioners
herein, we dismissed the petition ruling that the more appropriate remedy of the petitioners is a separate
proceeding for the probate of the will in question. Pursuant to the said ruling, the petitioners commenced
in the then Court of First Instance of Iloilo, Special Proceeding No. 2176, for the probate of the disputed
will, which was opposed by the private respondents presently, Panfilo and Felino both surnamed Maloto.
The trial court dismissed the petition on April 30, 1970. Complaining against the dismissal, again, the
petitioners came to this Court on a petition for review by certiorari. 2 Acting on the said petition, we set
aside the trial court's order and directed it to proceed to hear the case on the merits. The trial court, after
hearing, found the will to have already been revoked by the testatrix. Adriana Maloto, and thus, denied
the petition. The petitioners appealed the trial court's decision to the Intermediate Appellate Court which,
on June 7, 1985, affirmed the order. The petitioners' motion for reconsideration of the adverse decision
proved to be of no avail, hence, this petition.

For a better understanding of the controversy, a factual account would be a great help.

On October 20, 1963, Adriana Maloto died leaving as heirs her niece and nephews, the petitioners
Aldina Maloto-Casiano and Constancio, Maloto, and the private respondents Panfilo Maloto and
Felino Maloto. Believing that the deceased did not leave behind a last will and testament, these four
heirs commenced on November 4, 1963 an intestate proceeding for the settlement of their aunt's
estate. The case was instituted in the then Court of First Instance of Iloilo and was docketed as
Special Proceeding No. 1736. However, while the case was still in progress, or to be exact on
February 1, 1964, the parties Aldina, Constancio, Panfilo, and Felino executed an agreement
of extrajudicial settlement of Adriana's estate. The agreement provided for the division of the estate
into four equal parts among the parties. The Malotos then presented the extrajudicial settlement
agreement to the trial court for approval which the court did on March 21, 1964. That should have
signalled the end of the controversy, but, unfortunately, it had not.

Three years later, or sometime in March 1967, Atty. Sulpicio Palma, a former associate of Adriana's
counsel, the late Atty. Eliseo Hervas, discovered a document entitled "KATAPUSAN NGA
PAGBUBULAT-AN (Testamento)," dated January 3,1940, and purporting to be the last will and
testament of Adriana. Atty. Palma claimed to have found the testament, the original copy, while he
was going through some materials inside the cabinet drawer formerly used by Atty. Hervas. The
document was submitted to the office of the clerk of the Court of First Instance of Iloilo on April 1,
1967. Incidentally, while Panfilo and Felino are still named as heirs in the said will, Aldina and
Constancio are bequeathed much bigger and more valuable shares in the estate of Adriana than
what they received by virtue of the agreement of extrajudicial settlement they had earlier signed. The
will likewise gives devises and legacies to other parties, among them being the petitioners Asilo de
Molo, the Roman Catholic Church of Molo, and Purificacion Miraflor.
Thus, on May 24, 1967, Aldina and Constancio, joined by the other devisees and legatees named in
the will, filed in Special Proceeding No. 1736 a motion for reconsideration and annulment of the
proceedings therein and for the allowance of the will When the trial court denied their motion, the
petitioner came to us by way of a petition for certiorari and mandamus assailing the orders of the trial
court . 3 As we stated earlier, we dismissed that petition and advised that a separate proceeding for the
probate of the alleged will would be the appropriate vehicle to thresh out the matters raised by the
petitioners.

Significantly, the appellate court while finding as inconclusive the matter on whether or not the
document or papers allegedly burned by the househelp of Adriana, Guadalupe Maloto Vda. de
Coral, upon instructions of the testatrix, was indeed the will, contradicted itself and found that the will
had been revoked. The respondent court stated that the presence of animus revocandi in the
destruction of the will had, nevertheless, been sufficiently proven. The appellate court based its
finding on the facts that the document was not in the two safes in Adriana's residence, by the
testatrix going to the residence of Atty. Hervas to retrieve a copy of the will left in the latter's
possession, and, her seeking the services of Atty. Palma in order to have a new will drawn up. For
reasons shortly to be explained, we do not view such facts, even considered collectively, as
sufficient bases for the conclusion that Adriana Maloto's will had been effectively revoked.

There is no doubt as to the testamentary capacity of the testatrix and the due execution of the will.
The heart of the case lies on the issue as to whether or not the will was revoked by Adriana.

The provisions of the new Civil Code pertinent to the issue can be found in Article 830.

Art. 830. No will shall be revoked except in the following cases:

(1) By implication of law; or

(2) By some will, codicil, or other writing executed as provided in case of wills: or

(3) By burning, tearing, cancelling, or obliterating the will with the intention of
revoking it, by the testator himself, or by some other person in his presence, and by
his express direction. If burned, torn cancelled, or obliterated by some other person,
without the express direction of the testator, the will may still be established, and the
estate distributed in accordance therewith, if its contents, and due execution, and the
fact of its unauthorized destruction, cancellation, or obliteration are established
according to the Rules of Court. (Emphasis Supplied.)

It is clear that the physical act of destruction of a will, like burning in this case, does not per se
constitute an effective revocation, unless the destruction is coupled with animus revocandi on the
part of the testator. It is not imperative that the physical destruction be done by the testator himself. It
may be performed by another person but under the express direction and in the presence of the
testator. Of course, it goes without saying that the document destroyed must be the will itself.

In this case, while animus revocandi or the intention to revoke, may be conceded, for that is a state
of mind, yet that requisite alone would not suffice. "Animus revocandi is only one of the necessary
elements for the effective revocation of a last will and testament. The intention to revoke must be
accompanied by the overt physical act of burning, tearing, obliterating, or cancelling the will carried
out by the testator or by another person in his presence and under his express direction. There is
paucity of evidence to show compliance with these requirements. For one, the document or papers
burned by Adriana's maid, Guadalupe, was not satisfactorily established to be a will at all, much less
the will of Adriana Maloto. For another, the burning was not proven to have been done under the
express direction of Adriana. And then, the burning was not in her presence. Both witnesses,
Guadalupe and Eladio, were one in stating that they were the only ones present at the place where
the stove (presumably in the kitchen) was located in which the papers proffered as a will were
burned.

The respondent appellate court in assessing the evidence presented by the private respondents as
oppositors in the trial court, concluded that the testimony of the two witnesses who testified in favor
of the will's revocation appear "inconclusive." We share the same view. Nowhere in the records
before us does it appear that the two witnesses, Guadalupe Vda. de Corral and Eladio Itchon, both
illiterates, were unequivocably positive that the document burned was indeed Adriana's will.
Guadalupe, we think, believed that the papers she destroyed was the will only because, according to
her, Adriana told her so. Eladio, on the other hand, obtained his information that the burned
document was the will because Guadalupe told him so, thus, his testimony on this point is double
hearsay.

At this juncture, we reiterate that "(it) is an important matter of public interest that a purported win is
not denied legalization on dubious grounds. Otherwise, the very institution of testamentary
succession will be shaken to its very foundations ...." 4

The private respondents in their bid for the dismissal of the present action for probate instituted by
the petitioners argue that the same is already barred by res adjudicata. They claim that this bar was
brought about by the petitioners' failure to appeal timely from the order dated November 16, 1968 of
the trial court in the intestate proceeding (Special Proceeding No. 1736) denying their (petitioners')
motion to reopen the case, and their prayer to annul the previous proceedings therein and to allow
the last will and testament of the late Adriana Maloto. This is untenable.

The doctrine of res adjudicata finds no application in the present controversy. For a judgment to be a
bar to a subsequent case, the following requisites must concur: (1) the presence of a final former
judgment; (2) the former judgment was rendered by a court having jurisdiction over the subject
matter and the parties; (3) the former judgment is a judgment on the merits; and (4) there is,
between the first and the second action, Identity of parties, of subject matter, and of cause of
action. 5 We do not find here the presence of all the enumerated requisites.

For one, there is yet, strictly speaking, no final judgment rendered insofar as the probate of Adriana
Maloto's will is concerned. The decision of the trial court in Special Proceeding No. 1736, although
final, involved only the intestate settlement of the estate of Adriana. As such, that judgment could not
in any manner be construed to be final with respect to the probate of the subsequently discovered
will of the decedent. Neither is it a judgment on the merits of the action for probate. This is
understandably so because the trial court, in the intestate proceeding, was without jurisdiction to rule
on the probate of the contested will . 6 After all, an action for probate, as it implies, is founded on the
presence of a will and with the objective of proving its due execution and validity, something which can
not be properly done in an intestate settlement of estate proceeding which is predicated on the
assumption that the decedent left no will. Thus, there is likewise no Identity between the cause of action
in intestate proceeding and that in an action for probate. Be that as it may, it would be remembered that it
was precisely because of our ruling in G.R. No. L-30479 that the petitioners instituted this separate action
for the probate of the late Adriana Maloto's will. Hence, on these grounds alone, the position of the private
respondents on this score can not be sustained.

One last note. The private respondents point out that revocation could be inferred from the fact that
"(a) major and substantial bulk of the properties mentioned in the will had been disposed of: while an
insignificant portion of the properties remained at the time of death (of the testatrix); and,
furthermore, more valuable properties have been acquired after the execution of the will on January
3,1940." 7 Suffice it to state here that as these additional matters raised by the private respondents are
extraneous to this special proceeding, they could only be appropriately taken up after the will has been
duly probated and a certificate of its allowance issued.

WHEREFORE, judgment is hereby rendered REVERSING and SETTING ASIDE the Decision dated
June 7, 1985 and the Resolution dated October 22, 1986, of the respondent Court of Appeals, and a
new one ENTERED for the allowance of Adriana Maloto's last will and testament. Costs against the
private respondents.

This Decision is IMMEDIATELY EXECUTORY.

SO ORDERED.

Yap (Chairman), Melencio-Herrera, and Paras JJ., concur.

Padilla, J., took no part.

G.R. No. L-2538 September 21, 1951

Testate Estate of the Deceased MARIANO MOLO Y LEGASPI. JUANA JUAN VDA. DE
MOLO, petitioner-appellee,
vs.
LUZ, GLICERIA and CORNELIO MOLO, oppositors-appellants.

Claro M. Recto and Serafin C. Dizon for appellants.


Delgado & Flores for appellee.

BAUTISTA ANGELO, J.:

This is an appeal from an order of the Court of First Instance of Rizal admitting to probate the last
will and testament of the deceased Mariano Molo y Legaspi executed on August 17, 1918. The
oppositors-appellants brought the case on appeal to this Court for the reason that the value of the
properties involved exceeds P50,000.

Mariano Molo y Legaspi died on January 24, 1941, in the municipality of Pasay, province of Rizal,
without leaving any forced heir either in the descending or ascending line. He was survived,
however, by his wife, the herein petitioner Juana Juan Vda. de Molo, and by his nieces and nephew,
the oppositors-appellants, Luz Gliceria and Cornelio, all surnamed Molo, who were the legitimate
children of Candido Molo y Legaspi, deceased brother of the testator. Mariano Molo y Legaspi left
two wills, one executed on August 17, 1918, (Exhibit A) and another executed on June 20, 1939.
(Exhibit I). The later will executed in 1918.

On February 7, 1941, Juana Juan Vda. de Molo, filed in the Court of First Instance of Rizal a
petition, which was docketed as special proceeding No. 8022 seeking the probate of the will
executed by the deceased on June 20, 1939. There being no opposition, the will was probated.
However, upon petition filed by the herein oppositors, the order of the court admitting the will to
probate was set aside and the case was reopened. After hearing, at which both parties presented
their evidence, the court rendered decision denying the probate of said will on the ground that the
petitioner failed to prove that the same was executed in accordance with law.

In view of the disallowance of the will executed on June 20, 1939, the widow on February 24, 1944,
filed another petition for the probate of the will executed by the deceased on August 17, 1918, which
was docketed as special proceeding No. 56, in the same court. Again, the same oppositors filed an
opposition to the petition based on three grounds: (1) that petitioner is now estopped from seeking
the probate of the will of 1918; (2) that said will has not been executed in the manner required by law
and (3) that the will has been subsequently revoked. But before the second petition could be heard,
the battle for liberation came and the records of the case were destroyed. Consequently, a petition
for reconstitution was filed, but the same was found to be impossible because neither petitioner nor
oppositors could produce the copies required for its reconstitution. As a result, petitioner filed a new
petition on September 14, 1946, similar to the one destroyed, to which the oppositors filed an
opposition based on the same grounds as those contained in their former opposition. Then, the case
was set for trial, and on May 28, 1948, the court issued an order admitting the will to probate already
stated in the early part of this decision. From this order the oppositors appealed assigning six errors,
to wit.

I. The probate court erred in not holding that the present petitioner voluntarily and
deliberately frustrated the probate of the will dated June 20, 1939, in special proceeding No.
8022, in order to enable her to obtain the probate of another alleged will of Molo dated 191.

II. The court a quo erred in not holding that the petitioner is now estopped from seeking the
probate of Molo's alleged will of 1918.

III. The lower court erred in not holding that petitioner herein has come to court with "unclean
hands" and as such is not entitled to relief.

IV. The probate court erred in not holding that Molo's alleged will of August 17, 1918 was not
executed in the manner required by law.

V. The probate court erred in not holding that the alleged will of 1918 was deliberately
revoked by Molo himself.

VI. The lower court erred in not holding that Molo's will of 1918 was subsequently revoked by
the decedent's will of 1939.

In their first assignment of error, counsel for oppositors contend that the probate court erred in not
holding that the petitioner voluntarily and deliberately frustrated the probate of the will dated June 20,
1939, in order to enable her to obtain the probate of the will executed by the deceased on August
17, 1918, pointing out certain facts and circumstances with their opinion indicate that petitioner
connived with the witness Canuto Perez in an effort to defeat and frustrate the probate of the 1939
will because of her knowledge that said will intrinsically defective in that "the one and only
testamentory disposition thereof was a "disposicion captatoria". These circumstances, counsel for
the appellants contend, constitute a series of steps deliberately taken by petitioner with a view to
insuring the realization of her plan of securing the probate of the 1918 will which she believed would
better safeguard her right to inherit from the decease.

These imputations of fraud and bad faith allegedly committed in connection with special proceedings
No. 8022, now closed and terminated, are vigorously met by counsel for petitioner who contends
that to raise them in these proceedings which are entirely new and distinct and completely
independent from the other is improper and unfair as they find no support whatsoever in any
evidence submitted by the parties in this case. They are merely based on the presumptions and
conjectures not supported by any proof. For this reason, counsel, contends, the lower court was
justified in disregarding them and in passing them sub silentio in its decision.
A careful examination of the evidence available in this case seems to justify this contention. There is
indeed no evidence which may justify the insinuation that petitioner had deliberately intended to
frustrate the probate of the 1939 will of the deceased to enable her to seek the probate of another
will other than a mere conjecture drawn from the apparently unexpected testimony of Canuto Perez
that he went out of the room to answer an urgent call of nature when Artemio Reyes was signing the
will and the failure of petitioner later to impeach the character of said witness in spite of the
opportunity given her by the court to do so. Apart from this insufficiency of evidence, the record
discloses that this failure has been explained by petitioner when she informed the court that she was
unable to impeach the character of her witness Canuto Perez because of her inability to find
witnesses who may impeach him, and this explanation stands uncontradicted. Whether this
explanation is satisfactory or not, it is not now, for us to determine. It is an incident that comes within
the province of the former case. The failure of petitioner to present the testimony of Artemio Reyes
at the hearing has also been explained, and it appears that petitioner has filed because his
whereabouts could not be found. Whether this is true or not is also for this Court to determine. It is
likewise within the province and function of the court in the former case. And the unfairness of this
imputation becomes more glaring when we stock of the developments that had taken place in these
proceedings which show in bold relief the true nature of the conduct, behavior and character of the
petitioner so bitterly assailed and held in disrepute by the oppositors.

It should be recalled that the first petition for the probate of the will executed on June 20, 1939, was
filed on February 7, 1941, by the petitioner. There being no opposition, the will was probated.
Subsequently, however, upon petition of the herein oppositors, the order of the court admitting said
will to probate was set aside, over the vigorous opposition of the herein petitioner, and the case was
reopened. The reopening was ordered because of the strong opposition of the oppositors who
contended that he will had not been executed as required by law. After the evidence of both parties
had been presented, the oppositors filed an extensive memorandum wherein they reiterated their
view that the will should be denied probate. And on the strenght of this opposition, the court
disallowed the will.

If petitioner then knew that the 1939 will was inherently defective and would make the testamentary
disposition in her favor invalid and ineffective, because it is a "disposicion captatoria", which
knowledge she may easily acquire through consultation with a lawyer, there was no need her to go
through the order of filing the petition for the probate of the will. She could accomplish her desire by
merely suppressing the will or tearing or destroying it, and then take steps leading to the probate of
the will executed in 1918. But for her conscience was clear and bade her to take the only proper step
possible under the circumstances, which is to institute the necessary proceedings for the probate of
the 1939 will. This she did and the will was admitted to probate. But then the unexpected happened.
Over her vigorous opposition, the herein appellants filed a petition for reopening, and over her
vigorous objection, the same was granted and the case was reopened. Her motion for
reconsideration was denied. Is it her fault that the case was reopened? Is it her fault that the order
admitting the will to probate was set aside? That was a contingency which petitioner never expected.
Had appellants not filed their opposition to the probate of the will and had they limited their objection
to the intrinsic validity of said will, their plan to defeat the will and secure the intestacy of the
deceased would have perhaps been accomplished. But they failed in their strategy. If said will was
denied probate it is due to their own effort. It is now unfair to impute bad faith petitioner simply
because she exerted every effort to protect her own interest and prevent the intestacy of the
deceased to happen.

Having reached the foregoing conclusions, it is obvious that the court did not commit the second and
third errors imputed to it by the counsel for appellants. Indeed, petitioner cannot be considered guilty
or estoppel which would prevent her from seeking the probate of the 1918 will simply because of her
effort to obtain the allowance of the 1939 will has failed considering that in both the 1918 and 1939
wills she was in by her husband as his universal heir. Nor can she be charged with bad faith far
having done so because of her desire to prevent the intestacy of her husband. She cannot be
blamed being zealous in protecting her interest.

The next contention of appellants refers to the revocatory clause contained in 1939 will of the
deceased which was denied probate. They contend that, notwithstanding the disallowance of said
will, the revocatory clause is valid and still has the effect of nullifying the prior of 1918.

Counsel for petitioner meets this argument by invoking the doctrine laid down in the case of Samson
vs. Naval, (41 Phil., 838). He contends that the facts involved in that case are on all fours with the
facts of this case. Hence, the doctrine is that case is here controlling.

There is merit in this contention. We have carefully read the facts involved in the Samson case we
are indeed impressed by their striking similarity with the facts of this case. We do not need to recite
here what those facts are; it is enough to point out that they contain many points and circumstances
in common. No reason, therefore, is seen by the doctrine laid down in that case (which we quote
hereunder) should not apply and control the present case.

A subsequent will, containing a clause revoking a previous will, having been disallowed, for
the reason that it was not executed in conformity with the provisions of section 618 of the
Code of Civil Procedure as to the making of wills, cannot produce the effect of annulling the
previous will, inasmuch as said revocatory clause is void. (41 Phil., 838.)

Apropos of this question, counsel for oppositors make the remark that, while they do not disagree
with the soundness of the ruling laid down in the Samson case, there is reason to abandon said
ruling because it is archaic or antiquated and runs counter to the modern trend prevailing in
American jurisprudence. They maintain that said ruling is no longer controlling but merely represents
the point of view of the minority and should, therefore, be abandoned, more so if we consider the
fact that section 623 of our Code of Civil Procedure, which governs the revocation of wills, is of
American origin and as such should follow the prevailing trend of the majority view in the United
States. A long line of authorities is cited in support of this contention. And these authorities hold the
view, that "an express revocation is immediately effective upon the execution of the subsequent will,
and does not require that it first undergo the formality of a probate proceeding". (p. 63, appellants'
brief .

While they are many cases which uphold the view entertained by counsel for oppositors, and that
view appears to be in controlling the states where the decisions had been promulgated, however, we
are reluctant to fall in line with the assertion that is now the prevailing view in the United States. In
the search we have made of American authorities on the subject, we found ourselves in a pool of
conflicting opinions perhaps because of the peculiar provisions contained in the statutes adopted by
each State in the subject of revocation of wills. But the impression we gathered from a review and
the study of the pertinent authorities is that the doctrine laid down in the Samson case is still a good
law. On page 328 of the American Jurisprudence Vol. 57, which is a revision Published in 1948, we
found the following passages which in our opinion truly reflect the present trend of American
jurisprudence on this matter affecting the revocation of wills:

SEC. 471. Observance of Formalities in Execution of Instrument. Ordinarily, statutes


which permit the revocation of a will by another writing provide that to be effective as a
revocation, the writing must be executed with the same formalities which are required to be
observed in the execution of a will. Accordingly, where, under the statutes, attestation is
necessary to the making of a valid will, an unattested non testamentary writing is not
effective to revoke a prior will. It has been held that a writing fails as a revoking instrument
where it is not executed with the formalities requisite for the execution of a will, even though
it is inscribed on the will itself, although it may effect a revocation by cancellation or
obliteration of the words of the will. A testator cannot reserve to himself the power to modify
a will by a written instrument subsequently prepared but not executed in the manner required
for a will.

SEC, 472. Subsequent Unexecuted, Invalid, or Ineffective Will or Codicil. A will which is
invalid because of the incapacity of the testator, or of undue influence can have no effect
whatever as a revoking will. Moreover, a will is not revoked by the unexecuted draft of a later
one. Nor is a will revoked by a defectively executed will or codicil, even though the latter
contains a clause expressly revoking the former will, in a jurisdiction where it is provided by a
controlling statute that no writing other than a testamentary instrument is sufficient to revoke
a will, for the simple reason that there is no revoking will. Similarly where the statute provides
that a will may be revoked by a subsequent will or other writing executed with the same
formalities as are required in the execution of wills, a defectively executed will does not
revoke a prior will, since it cannot be said that there is a writing which complies with the
statute. Moreover, a will or codicil which, on account of the manner in which it is executed, is
sufficient to pass only personally does not affect dispositions of real estate made by a former
will, even though it may expressly purport to do so. The intent of the testator to revoke is
immaterial, if he has not complied with the statute. (57 Am. Jur., 328, 329.)

We find the same opinion in the American Law Reports, Annotated, edited in 1939. On page 1400,
Volume 123, there appear many authorities on the "application of rules where second will is invalid",
among which a typical one is the following:

It is universally agreed that where the second will is invalid on account of not being executed
in accordance with the provisions of the statute, or where the testator who has not sufficient
mental capacity to make a will or the will is procured through undue influence, or the such, in
other words, where the second will is really no will, it does not revoke the first will or affect it
in any manner. Mort vs. Baker University (193-5) 229 Mo. App., 632, 78 S.W. (2d), 498.

These treaties cannot be mistaken. They uphold the view on which the ruling in the Samson case is
predicated. They reflect the opinion that this ruling is sound and good and for this reason, we see no
justification for abondoning it as now suggested by counsel for the oppositors.

It is true that our law on the matter (sec. 623, Code Civil Procedure) provides that a will may be
some will, codicil, or other writing executed as proved in case of wills" but it cannot be said that the
1939 will should be regarded, not as a will within the meaning of said word, but as "other writing
executed as provided in the case of wills", simply because it was denied probate. And even if it be
regarded as any other writing within the meaning of said clause, there is authority for holding that
unless said writing is admitted to probate, it cannot have the effect of revocation. (See 57 Am. Jur.
pp. 329-330).

But counsel for oppositors contemned that, regardless of said revocatory clause, said will of 1918
cannot still be given effect because of the presumption that it was deliberately revoked by the
testator himself. The oppositors contend that the testator, after executing the 1939 will, and with full
knowledge of the recovatory clause contained said will, himself deliberately destroyed the original of
the 1918 will, and for that reason the will submitted by petitioner for probate in these proceedings is
only a duplicate of said original.

There is no evidence which may directly indicate that the testator deliberately destroyed the original
of the 1918 will because of his knowledge of the revocatory clause contained in the will he executed
in 1939. The only evidence we have is that when the first will was executed in 1918, Juan Salcedo,
who prepared it, gave the original and copies to the testator himself and apparently they remained in
his possession until he executed his second will in 1939. And when the 1939 will was denied probate
on November 29, 1943, and petitioner was asked by her attorney to look for another will, she found
the duplicate copy (Exhibit A) among the papers or files of the testator. She did not find the original.

If it can be inferred that the testator deliberately destroyed the 1918 will because of his knowledge of
the revocatory clause of the 1939 will, and it is true that he gave a duplicate copy thereof to his wife,
the herein petitioner, the most logical step for the testator to take is to recall said duplicate copy in
order that it may likewise be destroyed. But this was not done as shown by the fact that said
duplicate copy remained in the possession of petitioner. It is possible that because of the long lapse
of twenty-one (21) years since the first will was executed, the original of the will had been misplaced
or lost, and forgetting that there was a copy, the testator deemed it wise to execute another will
containing exactly the same testamentary dispositions. Whatever may be the conclusion we may
draw from this chain of circumstances, the stubborn fact is that there is no direct evidence of
voluntary or deliberate destruction of the first will by the testator. This matter cannot be inference or
conjectur.

Granting for the sake of argument that the earlier will was voluntarily destroyed by the testator after
the execution of the second will, which revoked the first, could there be any doubt, under this theory,
that said earlier will was destroyed by the testator in the honest belief that it was no longer
necessary because he had expressly revoked it in his will of 1939? In other words, can we not say
that the destruction of the earlier will was but the necessary consequence of the testator's belief that
the revocatory clause contained in the subsequent will was valid and the latter would be given
effect? If such is the case, then it is our opinion that the earlier will can still be admitted to probate
under the principle of "dependent relative revocation".

This doctrine is known as that of dependent relative revocation, and is usually applied where
the testator cancels or destroys a will or executes an instrument intended to revoke a will
with a present intention to make a new testamentary disposition as a substitute for the old,
and the new disposition is not made or, if made, fails of effect for same reason. The doctrine
is n limited to the existence of some other document, however, and has been applied where
a will was destroyed as a consequence of a mistake of law. . . . (68 C.J.P. 799).

The rule is established that where the act of destruction is connected with the making of
another will so as fairly to raise the inference that the testator meant the revocation of the old
to depend upon the efficacy of a new disposition intended to be substituted, the revocation
will be conditional and dependent upon the efficacy of the new disposition; and if, for any
reason, the new will intended to be made as a substitute is inoperative, the revocation fails
and the original will remains in full force. (Gardner, pp. 232, 233.)

This is the doctrine of dependent relative revocation. The failure of a new testamentary
disposition upon whose validity the revocation depends, is equivalent to the non-fulfillment of
a suspensive conditions, and hence prevents the revocation of the original will. But a mere
intent to make at some time a will in the place of that destroyed will not render the
destruction conditional. It must appear that the revocation is dependent upon the valid
execution of a new will. (1 Alexander, p. 751; Gardner, p. 253.)

We hold therefore, that even in the supposition that the destruction of the original will by the testator
could be presumed from the failure of the petitioner to produce it in court, such destruction cannot
have the effect of defeating the prior will of 1918 because of the fact that it is founded on the
mistaken belief that the will of 1939 has been validly executed and would be given due effect. The
theory on which this principle is predicated is that the testator did not intend to die intestate. And this
intention is clearly manifest when he executed two wills on two different occasion and instituted his
wife as his universal heir. There can therefore be no mistake as to his intention of dying testate.

The remaining question to be determined refers to the sufficiency of the evidence to prove the due
execution of the will.

The will in question was attested, as required by law, by three witnesses, Lorenzo Morales, Rufino
Enriquez, and Angel Cuenca. The first two witnesses died before the commencement of the present
proceedings. So the only instrumental witness available was Angel Cuenca and under our law and
precedents, his testimony is sufficient to prove the due execution of the will. However, petitioner
presented not only the testimony of Cuenca but placed on the witness stand Juan Salcedo, the
notary public who prepared and notarized the will upon the express desire and instruction of the
testator, The testimony of these witnesses shows that the will had been executed in the manner
required by law. We have read their testimony and we were impressed by their readiness and
sincerity. We are convinced that they told the truth.

Wherefore, the order appealed from is hereby affirmed, with costs against the appellants. 1wp

Paras, C.J. Feria, Pablo Bengzon, Tuason and Jugo JJ., concur.

G.R. No. L-58509 December 7, 1982

IN THE MATTER OF THE PETITION TO APPROVE THE WILL OF RICARDO B. BONILLA


deceased, MARCELA RODELAS, petitioner-appellant,
vs.
AMPARO ARANZA, ET AL., oppositors-appellees, ATTY. LORENZO SUMULONG, intervenor.

Luciano A. Joson for petitioner-appellant.

Cesar Paralejo for oppositor-appellee.

RELOVA, J.:

This case was certified to this Tribunal by the Court of Appeals for final determination pursuant to
Section 3, Rule 50 of the Rules of Court.

As found by the Court of Appeals:

... On January 11, 1977, appellant filed a petition with the Court of First Instance of
Rizal for the probate of the holographic will of Ricardo B. Bonilla and the issuance of
letters testamentary in her favor. The petition, docketed as Sp. Proc. No. 8432, was
opposed by the appellees Amparo Aranza Bonilla, Wilferine Bonilla Treyes Expedita
Bonilla Frias and Ephraim Bonilla on the following grounds:
(1) Appellant was estopped from claiming that the deceased left a will by failing to
produce the will within twenty days of the death of the testator as required by Rule
75, section 2 of the Rules of Court;

(2) The alleged copy of the alleged holographic will did not contain a disposition of
property after death and was not intended to take effect after death, and therefore it
was not a will

(3) The alleged hollographic will itself,and not an alleged copy thereof, must be
produced, otherwise it would produce no effect, as held in Gam v. Yap, 104 Phil.
509; and

(4 ) The deceased did not leave any will, holographic or otherwise, executed and
attested as required by law.

The appellees likewise moved for the consolidation of the case with another case Sp.
Proc. No, 8275). Their motion was granted by the court in an order dated April 4,
1977.

On November 13, 1978, following the consolidation of the cases, the appellees
moved again to dismiss the petition for the probate of the will. They argued that:

(1) The alleged holographic was not a last will but merely an instruction as to the
management and improvement of the schools and colleges founded by decedent
Ricardo B. Bonilla; and

(2) Lost or destroyed holographic wills cannot be proved by secondary evidence


unlike ordinary wills.

Upon opposition of the appellant, the motion to dismiss was denied by the court in its
order of February 23, 1979.

The appellees then filed a motion for reconsideration on the ground that the order
was contrary to law and settled pronouncements and rulings of the Supreme Court,
to which the appellant in turn filed an opposition. On July 23, 1979, the court set
aside its order of February 23, 1979 and dismissed the petition for the probate of the
will of Ricardo B. Bonilla. The court said:

... It is our considered opinion that once the original copy of the holographic will is
lost, a copy thereof cannot stand in lieu of the original.

In the case of Gam vs. Yap, 104 Phil. 509, 522, the Supreme Court held that 'in the
matter of holographic wills the law, it is reasonable to suppose, regards the
document itself as the material proof of authenticity of said wills.

MOREOVER, this Court notes that the alleged holographic will was executed on
January 25, 1962 while Ricardo B. Bonilla died on May 13, 1976. In view of the lapse
of more than 14 years from the time of the execution of the will to the death of the
decedent, the fact that the original of the will could not be located shows to our mind
that the decedent had discarded before his death his allegedly missing Holographic
Will.
Appellant's motion for reconsideration was denied. Hence, an appeal to the Court of Appeals in
which it is contended that the dismissal of appellant's petition is contrary to law and well-settled
jurisprudence.

On July 7, 1980, appellees moved to forward the case to this Court on the ground that the appeal
does not involve question of fact and alleged that the trial court committed the following assigned
errors:

I. THE LOWER COURT ERRED IN HOLDING THAT A LOST HOLOGRAPHIC WILL


MAY NOT BE PROVED BY A COPY THEREOF;

II. THE LOWER COURT ERRED IN HOLDING THAT THE DECEDENT HAS
DISCARDED BEFORE HIS DEATH THE MISSING HOLOGRAPHIC WILL;

III. THE LOWER COURT ERRED IN DISMISSING APPELLANT'S WILL.

The only question here is whether a holographic will which was lost or cannot be found can be
proved by means of a photostatic copy. Pursuant to Article 811 of the Civil Code, probate of
holographic wills is the allowance of the will by the court after its due execution has been proved.
The probate may be uncontested or not. If uncontested, at least one Identifying witness is required
and, if no witness is available, experts may be resorted to. If contested, at least three Identifying
witnesses are required. However, if the holographic will has been lost or destroyed and no other
copy is available, the will can not be probated because the best and only evidence is the handwriting
of the testator in said will. It is necessary that there be a comparison between sample handwritten
statements of the testator and the handwritten will. But, a photostatic copy or xerox copy of the
holographic will may be allowed because comparison can be made with the standard writings of the
testator. In the case of Gam vs. Yap, 104 PHIL. 509, the Court ruled that "the execution and the
contents of a lost or destroyed holographic will may not be proved by the bare testimony of
witnesses who have seen and/or read such will. The will itself must be presented; otherwise, it shall
produce no effect. The law regards the document itself as material proof of authenticity." But, in
Footnote 8 of said decision, it says that "Perhaps it may be proved by a photographic or photostatic
copy. Even a mimeographed or carbon copy; or by other similar means, if any, whereby the
authenticity of the handwriting of the deceased may be exhibited and tested before the probate
court," Evidently, the photostatic or xerox copy of the lost or destroyed holographic will may be
admitted because then the authenticity of the handwriting of the deceased can be determined by the
probate court.

WHEREFORE, the order of the lower court dated October 3, 1979, denying appellant's motion for
reconsideration dated August 9, 1979, of the Order dated July 23, 1979, dismissing her petition to
approve the will of the late Ricardo B. Bonilla, is hereby SET ASIDE.

SO ORDERED.

Teehankee, Actg. C.J., Melencio-Herrera, Plana, Vasquez and Gutierrez, Jr., JJ., concur.

G.R. No. L-14003 August 5, 1960


FEDERICO AZAOLA, petitioner-appellant,
vs.
CESARIO SINGSON, oppositor-appellee.

F. Lavides and L.B. Alcuaz for appellant.


Vicente J. Cuna and P.S. Singson for appellee.

REYES, J.B.L., J.:

This appeal, taken on points of law from a decision rendered on 15 January 1958 by the Court of
First Instance of Quezon City in its Special Proceedings No. Q-2640, involves the determination of
the quantity of evidence required for the probate of a holographic will.

The established facts are thus summarized in the decision appealed from (Rec. App. pp. 22-24):

"Briefly speaking, the following facts were established by the petitioner; that on September 9,
1957, Fortunata S. Vda. de Yance died at 13 Luskot, Quezon City, known to be the last
residence of said testatrix; that Francisco Azaola, petitioner herein for probate of the
holographic will, submitted the said holographic will (Exh. C) whereby Maria Milagros Azaola
was made the sole heir as against the nephew of deceased Cesario Singson; that witness
Francisco Azaola testified that he saw the holographic will (Exh. C) one month, more or less,
before the death of the testatrix, as the same was handed to him and his wife; that the
witness testified also that he recognized all the signatures appearing in the holographic will
(Exh. C) as the handwriting of the testatrix and to reinforce said statement, witness
presented the mortgage (Exh. E), the special power of the attorney (Exh. F), and the general
power of attorney (Exh. F-1), besides the deeds of sale (Exhs. G and G-1) including an
affidavit (Exh. G-2), and that there were further exhibited in court two residence certificates
(Exhs. H and H-1) to show the signatures of the testatrix, for comparison purposes; that said
witness, Azaola, testified that the penmanship appearing in the aforesaid documentary
evidence is in the handwriting of the testatrix as well as the signatures appearing in the
aforesaid documentary evidence is in the handwriting of the testatrix as well as the
signatures appearing therein are the signatures of the testatrix; that said witness, in answer
to a question of his counsel admitted that the holographic will was handed to him by the
testatrix. "apparently it must have been written by her" (t.s.n., p. 11). However, on page 16
on the same transcript of the stenographic notes, when the same witness was asked by
counsel if he was familiar with the penmanship and handwriting of the deceased Fortunata
Vda. de Yance, he answered positively in the affirmative and when he was asked again
whether the penmanship referred to in the previous answer as appearing in the holographic
will (Exh. C) was hers (testatrix'), he answered, "I would definitely say it is hers"; that it was
also established in the proceedings that the assessed value of the property of the deceased
in Luskot, Quezon City, is in the amount of P7,000.00.

The opposition to the probate was on the ground that (1) the execution of the will was procured by
undue and improper pressure and influence on the part of the petitioner and his wife, and (2) that the
testatrix did not seriously intend the instrument to be her last will, and that the same was actually
written either on the 5th or 6th day of August 1957 and not on November 20, 1956 as appears on the
will.

The probate was denied on the ground that under Article 811 of the Civil Code, the proponent must
present three witnesses who could declare that the will and the signature are in the writing of the
testatrix, the probate being contested; and because the lone witness presented by the proponent
"did not prove sufficiently that the body of the will was written in the handwriting of the testatrix."
The proponent appealed, urging: first, that he was not bound to produce more than one witness
because the will's authenticity was not questioned; and second, that Article 811 does not mandatorily
require the production of three witnesses to identify the handwriting and signature of a holographic
will, even if its authenticity should be denied by the adverse party.

Article 811 of the Civil Code of the Philippines is to the following effect:

ART. 811. In the probate of a holographic will, it shall be necessary that at least one witness
who knows the handwriting and signature of the testator explicitly declare that the will and
the signature are in the handwriting of the testator. If the will is contested, at least three of
such witnesses shall be required.

In the absence of any competent witnesses referred to in the preceding paragraph, and if the
court deems it necessary, expert testimony may be resorted to. (691a).

We agree with the appellant that since the authenticity of the will was not contested, he was not
required to produce more than one witness; but even if the genuineness of the holographic will were
contested, we are of the opinion that Article 811 of our present Civil Code can not be interpreted as
to require the compulsory presentation of three witnesses to identify the handwriting of the testator,
under penalty of having the probate denied. Since no witness may have been present at the
execution of a holographic will, none being required by law (Art. 810, new Civil Code), it becomes
obvious that the existence of witness possessing the requisite qualifications is a matter beyond the
control of the proponent. For it is not merely a question of finding and producing any three
witnesses; they must be witnesses "who know the handwriting and signature of the testator" and
who can declare (truthfully, of course, even if the law does not so express) "that the will and the
signature are in the handwriting of the testator". There may be no available witness of the testator's
hand; or even if so familiarized, the witnesses may be unwilling to give a positive opinion.
Compliance with the rule of paragraph 1 of Article 811 may thus become an impossibility. That is
evidently the reason why the second paragraph of Article 811 prescribes that

in the absence of any competent witness referred to in the preceding paragraph, and if the
court deems it necessary, expert testimony may be resorted to.

As can be seen, the law foresees the possibility that no qualified witness may be found (or what
amounts to the same thing, that no competent witness may be willing to testify to the authenticity of
the will), and provides for resort to expert evidence to supply the deficiency.

It may be true that the rule of this article (requiring that three witnesses be presented if the will is
contested and only one if no contest is had) was derived from the rule established for ordinary
testaments (cf. Cabang vs. Delfinado, 45 Phil., 291; Tolentino vs. Francisco, 57 Phil., 742). But it can
not be ignored that the requirement can be considered mandatory only in the case of ordinary
testaments, precisely because the presence of at least three witnesses at the execution of ordinary
wills is made by law essential to their validity (Art. 805). Where the will is holographic, no witness
need be present (Art. 10), and the rule requiring production of three witnesses must be deemed
merely permissive if absurd results are to be avoided.

Again, under Article 811, the resort to expert evidence is conditioned by the words "if the Court deem
it necessary", which reveal that what the law deems essential is that the Court should be convinced
of the will's authenticity. Where the prescribed number of witnesses is produced and the court is
convinced by their testimony that the ill is genuine, it may consider it unnecessary to call for expert
evidence. On the other hand, if no competent witness is available, or none of those produced is
convincing, the Court may still, and in fact it should, resort to handwriting experts. The duty of the
Court, in fine, is to exhaust all available lines of inquiry, for the state is as much interested as the
proponent that the true intention of the testator be carried into effect.

Commenting on analogous provisions of Article 691 of the Spanish Civil Code of 1889, the noted
Commentator, Mucuis Scaevola (Vol. 12, 2nd Ed., p.421), sagely remarks:

La manera como esta concebida la redaccion del ultimo apartado de dicho precepto induce
la conclusion de que siempre o por lo menos, en la mayor parte de los casos, el Juez debe
acudir al criterio pericial para que le ilustre acerca de la autenticidad del testamento olografo,
aunque ya esten insertas en los autos del expediente las declaraciones testificales. La
prudencia con que el Juez debe de proceder en resoluciones de transcendencia asi lo exige,
y la indole delicada y peligrosa del testamento olografo lo hace necesario para mayor
garantia de todos los interes comprometidos en aquel.

En efecto, el cotejo pericial de letras puede ser una confirmacion facultativa del dicho
profano de los testigos y un modo de desvanecer las ultimas dudas que pudieran ocurrir al
Juez acerca de la autenticidad que trata de averigaur y declarar. Para eso se ha escrito la
frase del citado ultimo apartado, (siempre que el Juez lo estime conveniente), haya habido o
no testigos y dudaran o no estos respecto de los extremos por que son preguntados.

El arbitrio judicial en este caso debe formarse con independencia de los sucesos y de su
significacion, para responder debidamente de las resoluciones que haya de dictar.

And because the law leaves it to the trial court if experts are still needed, no unfavourable inference
can be drawn from a party's failure to offer expert evidence, until and unless the court expresses
dissatisfaction with the testimony of the lay witnesses.

Our conclusion is that the rule of the first paragraph of Article 811 of the Civil Code is merely
directory and is not mandatory.

Considering, however, that this is the first occasion in which this Court has been called upon to
construe the import of said article, the interest of justice would be better served, in our opinion, by
giving the parties ample opportunity to adduce additional evidence, including expert witnesses,
should the Court deem them necessary.

In view of the foregoing, the decision appealed from is set aside, and the records ordered remanded
to the Court of origin, with instructions to hold a new trial in conformity with this opinion. But evidence
already on record shall not be retaken. No costs.

Bengzon, Padilla, Bautista Angelo, Labrador, Concepcion, Barrera and Gutierrez David, JJ., concur.

G.R. No. L-24742 October 26, 1973

ROSA CAYETANO CUENCO, petitioners,


vs.
THE HONORABLE COURT OF APPEALS, THIRD DIVISION, MANUEL CUENCO, LOURDES
CUENCO, CONCEPCION CUENCO MANGUERRA, CARMEN CUENCO, CONSUELO CUENCO
REYES, and TERESITA CUENCO GONZALEZ, respondents.

Ambrosio Padilla Law Office for petitioner.


Jalandoni and Jamir for respondents.

TEEHANKEE, J.:

Petition for certiorari to review the decision of respondent Court of Appeals in CA-G.R. No. 34104-R,
promulgated 21 November 1964, and its subsequent Resolution promulgated 8 July 1964 denying
petitioner's Motion for Reconsideration.

The pertinent facts which gave rise to the herein petition follow:

On 25 February 1964 Senator Mariano Jesus Cuenco died at the Manila Doctors' Hospital, Manila.
He was survived by his widow, the herein petitioner, and their two (2) minor sons, Mariano Jesus, Jr.
and Jesus Salvador, both surnamed Cuenco, all residing at 69 Pi y Margal St., Sta. Mesa Heights,
Quezon City, and by his children of the first marriage, respondents herein, namely, Manuel Cuenco,
Lourdes Cuenco, Concepcion Cuenco Manguera, Carmen Cuenco, Consuelo Cuenco Reyes and
Teresita Cuenco Gonzales, all of legal age and residing in Cebu.

On 5 March 1964, (the 9th day after the death of the late Senator) 1 respondent Lourdes Cuenco filed
a Petition for Letters of Administration with the court of first instance of Cebu (Sp. Proc. No. 2433-R),
alleging among other things, that the late senator died intestate in Manila on 25 February 1964; that he
was a resident of Cebu at the time of his death; and that he left real and personal properties in Cebu and
Quezon City. On the same date, the Cebu court issued an order setting the petition for hearing on 10
April 1964, directing that due notice be given to all the heirs and interested persons, and ordering the
requisite publication thereof at LA PRENSA, a newspaper of general circulation in the City and Province
of Cebu.

The aforesaid order, however, was later suspended and cancelled and a new and modified one
released on 13 March 1964, in view of the fact that the petition was to be heard at Branch II instead
of Branch I of the said Cebu court. On the same date, a third order was further issued stating that
respondent Lourdes Cuenco's petition for the appointment of a special administrator dated 4 March
1964 was not yet ready for the consideration of the said court, giving as reasons the following:

It will be premature for this Court to act thereon, it not having yet regularly acquired
jurisdiction to try this proceeding, the requisite publication of the notice of hearing not
yet having been complied with. Moreover, copies of the petition have not been
served on all of the heirs specified in the basic petition for the issuance of letters of
administration. 2

In the meantime, or specifically on 12 March 1964, (a week after the filing of the Cebu petition)
herein petitioner Rosa Cayetano Cuenco filed a petition with the court of first instance of Rizal
(Quezon City) for the probate of the deceased's last will and testament and for the issuance of letters
testamentary in her favor, as the surviving widow and executrix in the said last will and testament.
The said proceeding was docketed as Special Proceeding No. Q-7898.

Having learned of the intestate proceeding in the Cebu court, petitioner Rosa Cayetano Cuenco filed
in said Cebu court an Opposition and Motion to Dismiss, dated 30 March 1964, as well as an
Opposition to Petition for Appointment of Special Administrator, dated 8 April 1964. On 10 April
1964, the Cebu court issued an order holding in abeyance its resolution on petitioner's motion to
dismiss "until after the Court of First Instance of Quezon City shall have acted on the petition
for probate of that document purporting to be the last will and testament of the deceased Don
Mariano Jesus Cuenco." 3 Such order of the Cebu court deferring to the probateproceedings in the
Quezon City court was neither excepted to nor sought by respondents to be reconsidered or set aside by
the Cebu court nor did they challenge the same by certiorari or prohibition proceedings in the appellate
courts.

Instead, respondents filed in the Quezon City court an Opposition and Motion to Dismiss, dated 10
April 1964,opposing probate of the will and assailing the jurisdiction of the said Quezon City court to
entertain petitioner's petition for probate and for appointment as executrix in Sp. Proc. No. Q-7898 in
view of the alleged exclusive jurisdiction vested by her petition in the Cebu court in Sp. Proc. No.
2433-R. Said respondent prayed that Sp. Proc. No. Q-7898 be dismissed for lack of
jurisdiction and/or improper venue.

In its order of 11 April 1964, the Quezon City court denied the motion to dismiss, giving as a principal
reason the "precedence of probate proceeding over an intestate proceeding." 4 The said court further
found in said order that theresidence of the late senator at the time of his death was at No. 69 Pi y
Margal, Sta. Mesa Heights, Quezon City. The pertinent portion of said order follows:

On the question of residence of the decedent, paragraph 5 of the opposition and


motion to dismiss reads as follows: "that since the decedent Don Mariano Jesus
Cuenco was a resident of the City of Cebu at the time of his death, the aforesaid
petition filed by Rosa Cayetano Cuenco on 12 March 1964 was not filed with the
proper Court (wrong venue) in view of the provisions of Section 1 of Rule 73 of the
New Rules of Court ...". From the aforequoted allegation, the Court is made to
understand that the oppositors do not mean to say that the decedent being a resident
of Cebu City when he died, the intestate proceedings in Cebu City should prevail
over the probate proceedings in Quezon City, because as stated above the probate
of the will should take precedence, but that the probate proceedings should be filed
in the Cebu City Court of First Instance. If the last proposition is the desire of the
oppositors as understood by this Court, that could not also be entertained as proper
because paragraph 1 of the petition for the probate of the will indicates that Don
Mariano Jesus Cuenco at the time of his death was a resident of Quezon City at 69
Pi y Margal. Annex A (Last Will and Testament of Mariano Jesus Cuenco) of the
petition for probate of the will shows that the decedent at the time when he executed
his Last Will clearly stated that he is a resident of 69 Pi y Margal, Sta. Mesa Heights,
Quezon City, and also of the City of Cebu. He made the former as his first choice
and the latter as his second choice of residence." If a party has two residences, the
one will be deemed or presumed to his domicile which he himself selects or
considers to be his home or which appears to be the center of his affairs. The
petitioner, in thus filing the instant petition before this Court, follows the first choice of
residence of the decedent and once this court acquires jurisdiction of the probate
proceeding it is to the exclusion of all others. 5

Respondent Lourdes Cuenco's motion for reconsideration of the Quezon City court's said order of 11
April 1964 asserting its exclusive jurisdiction over the probate proceeding as deferred to by the Cebu
court was denied on 27 April 1964 and a second motion for reconsideration dated 20 May 1964 was
likewise denied.

On 11 May 1964, pursuant to its earlier order of 11 April 1964, the hearing for probate of the last will
of the decedent was called three times at half-hour intervals, but notwithstanding due notification
none of the oppositors appeared and the Quezon City court proceeded at 9:00 a.m. with the hearing
in their absence.
As per the order issued by it subsequently on 15 May 1964, the Quezon City court noted that
respondents-oppositors had opposed probate under their opposition and motion to dismiss on the
following grounds:

(a) That the will was not executed and attested as required by law;

(b) That the will was procured by undue and improper pressure and influence on the
part of the beneficiary or some other persons for his benefit;

(c) That the testator's signature was procured by fraud and/or that the testator acted
by mistake and did not intend that the instrument he signed should be his will at the
time he affixed his signature thereto. 6

The Quezon City court further noted that the requisite publication of the notice of the hearing had
been duly complied with and that all the heirs had been duly notified of the hearing, and after
receiving the testimony of the three instrumental witnesses to the decedent's last will, namely Atty.
Florencio Albino, Dr. Guillermo A. Picache and Dr. Jose P. Ojeda, and of the notary public, Atty.
Braulio A. Arriola, Jr., who ratified the said last will, and the documentary evidence (such as the
decedent's residence certificates, income tax return, diplomatic passport, deed of donation) all
indicating that the decedent was a resident of 69 Pi y Margal St., Quezon City, as also affirmed by
him in his last will, the Quezon City court in its said order of 15 May 1964 admitted to probate the
late senator's last will and testament as having been "freely and voluntarily executed by the testator"
and "with all formalities of the law" and appointed petitioner-widow as executrix of his estate without
bond "following the desire of the testator" in his will as probated.

Instead of appealing from the Quezon City court's said order admitting the will to probate and
naming petitioner-widow as executrix thereof, respondents filed a special civil action of certiorari and
prohibition with preliminary injunction with respondent Court of Appeals (docketed as case CA-G.R.
No. 34104-R) to bar the Rizal court from proceeding with case No. Q-7898.

On 21 November 1964, the Court of Appeals rendered a decision in favor of respondents (petitioners
therein) and against the herein petitioner, holding that:

Section 1, Rule 73, which fixes the venue in proceedings for the settlement of the
estate of a deceased person, covers both testate and intestate proceedings. Sp.
Proc. 2433-R of the Cebu CFI having been filed ahead, it is that court whose
jurisdiction was first invoked and which first attached. It is that court which can
properly and exclusively pass upon the factual issues of (1) whether the decedent left
or did not leave a valid will, and (2) whether or not the decedent was a resident of
Cebu at the time of his death.

Considering therefore that the first proceeding was instituted in the Cebu CFI
(Special Proceeding 2433-R), it follows that the said court must exercise jurisdiction
to the exclusion of the Rizal CFI, in which the petition for probate was filed by the
respondent Rosa Cayetano Cuenco (Special Proceeding Q-7898). The said
respondent should assert her rights within the framework of the proceeding in the
Cebu CFI, instead of invoking the jurisdiction of another court.

The respondents try to make capital of the fact that on March 13, 1964, Judge
Amador Gomez of the Cebu CFI, acting in Sp. Proc. 2433-R, stated that the petition
for appointment of special administrator was "not yet ready for the consideration of
the Court today. It would be premature for this Court to act thereon, it not having yet
regularly acquired jurisdiction to try this proceeding ... . " It is sufficient to state in this
connection that the said judge was certainly not referring to the court's jurisdiction
over the res, not to jurisdiction itself which is acquired from the moment a petition is
filed, but only to theexercise of jurisdiction in relation to the stage of the proceedings.
At all events, jurisdiction is conferred and determined by law and does not depend on
the pronouncements of a trial judge.

The dispositive part of respondent appellate court's judgment provided as follows:

ACCORDINGLY, the writ of prohibition will issue, commanding and directing the
respondent Court of First Instance of Rizal, Branch IX, Quezon City, and the
respondent Judge Damaso B. Tengco to refrain perpetually from proceeding and
taking any action in Special Proceeding Q-7898 pending before the said respondent
court. All orders heretofore issued and actions heretofore taken by said respondent
court and respondent Judge, therein and connected therewith, are hereby annulled.
The writ of injunction heretofore issued is hereby made permanent. No
pronouncement as to costs.

Petitioner's motion for reconsideration was denied in a resolution of respondent Court of Appeals,
dated 8 July 1965; hence the herein petition for review on certiorari.

The principal and decisive issue at bar is, theretofore, whether the appellate court erred in law in
issuing the writ of prohibition against the Quezon City court ordering it to refrain perpetually from
proceeding with the testateproceedings and annulling and setting aside all its orders and actions,
particularly its admission to probate of the decedent's last will and testament and appointing
petitioner-widow as executrix thereof without bond in compliance with the testator's express wish in
his testament. This issue is tied up with the issue submitted to the appellate court, to wit, whether the
Quezon City court acted without jurisdiction or with grave abuse of discretion in taking cognizance
and assuming exclusive jurisdiction over the probate proceedings filed with it, in pursuance of the
Cebu court's order of 10 April 1964 expressly consenting in deference to the precedence of probate
over intestate proceedings that it (the Quezon City court) should first act "on the petition for probate
of the document purporting to be the last will and testament of the deceased Don Mariano Jesus
Cuenco" - which order of the Cebu court respondents never questioned nor challenged by
prohibition or certiorari proceedings and thus enabled the Quezon City court to proceed without any
impediment or obstruction, once it denied respondent Lourdes Cuenco's motion to dismiss the
probate proceeding for alleged lack of jurisdiction or improper venue, toproceed with the hearing of
the petition and to admit the will to probate upon having been satisfied as to its due execution and
authenticity.

The Court finds under the above-cited facts that the appellate court erred in law in issuing the writ of
prohibition against the Quezon City court from proceeding with the testate proceedings and annulling
and setting aside all its orders and actions, particularly its admission to probate of the deceased's
last will and testament and appointing petitioner-widow as executrix thereof without bond pursuant to
the deceased testator's express wish, for the following considerations:

1. The Judiciary Act 7 concededly confers original jurisdiction upon all Courts of First Instance over "all
matter of probate, both of testate and intestate estates." On the other hand, Rule 73, section of the Rules
of Court lays down the rule of venue, as the very caption of the Rule indicates, and in order to prevent
conflict among the different courts which otherwise may properly assume jurisdiction from doing so, the
Rule specifies that "the court first taking cognizance of the settlement of the estate of a decedent,
shall exercise jurisdiction to the exclusion of all other courts." The cited Rule provides:
Section 1. Where estate of deceased persons settled. If the decedent is an inhabitant
of the Philippines at the time of his death, whether a citizen or an alien, his will shall
be proved, or letters of administration granted, and his estate settled, in the Court of
First Instance in the Province in which he resides at the time of his death, and if he is
an inhabitant of a foreign country, the Court of First Instance of the province in which
he had estate. The court first taking cognizance of the settlement of the estate of a
decedent, shall exercise jurisdiction to the exclusion of all other courts.
The jurisdiction assumed by a court, so far as it depends on the place of residence,
of the decedent, or of the location of his estate, shall not be contested in a suit or
proceeding, except in an appeal from that court, in the original case, or when
the want of jurisdiction appears on the record. (Rule 73) 8

It is equally conceded that the residence of the deceased or the location of his estate is not an
element of jurisdiction over the subject matter but merely of venue. This was lucidly stated by the
late Chief Justice Moran inSy Oa vs. Co Ho 9 as follows:

We are not unaware of existing decisions to the effect that in probate cases the place
of residence of the deceased is regarded as a question of jurisdiction over the
subject-matter. But we decline to follow this view because of its mischievous
consequences. For instance, a probate case has been submitted in good faith to the
Court of First Instance of a province where the deceased had not resided. All the
parties, however, including all the creditors, have submitted themselves to the
jurisdiction of the court and the case is therein completely finished except for a claim
of a creditor who also voluntarily filed it with said court but on appeal from an adverse
decision raises for the first time in this Court the question of jurisdiction of the trial
court for lack of residence of the deceased in the province. If we consider such
question of residence as one affecting the jurisdiction of the trial court over the
subject-matter, the effect shall be that the whole proceedings including all
decisions on the different incidents which have arisen in court will have to
be annulled and the same case will have to be commenced
anew before another court of the same rank in another province. That this is
ofmischievous effect in the prompt administration of justice is too obvious to require
comment. (Cf. Tanunchuan vs. Dy Buncio & Co., G.R. No. 48206, December 31,
1942) Furthermore, section 600 of Act No. 190, 10 providing that the estate of a
deceased person shall be settled in the province where he had last resided,
could not have been intended as defining the jurisdiction of the probate court over the
subject-matter, because such legal provision is contained in a law of procedure dealing
merely with procedural matters, and, as we have said time and again, procedure is one
thing and jurisdiction over the subject matter is another. (Attorney-General vs. Manila
Railroad Company, 20 Phil. 523.) The law of jurisdiction Act No. 136, 11 Section 56,
No. 5 confers upon Courts of First Instance jurisdiction over all probate cases
independently of the place of residence of the deceased. Since, however, there are many
courts of First Instance in the Philippines, the Law of Procedure, Act No. 190, section
600, fixes the venue or the place where each case shall be brought. Thus, the place
of residence of the deceased is not an element of jurisdiction over the subject-matter
but merely of venue. And it is upon this ground that in the new Rules of Court the
province where the estate of a deceased person shall be settled is properly called
"venue".

It should be noted that the Rule on venue does not state that the court with whom the estate or
intestate petition is first filed acquires exclusive jurisdiction.
The Rule precisely and deliberately provides that "the court first taking cognizance of the settlement
of the estateof a decedent, shall exercise jurisdiction to the exclusion of all other courts."

A fair reading of the Rule since it deals with venue and comity between courts of equal and co-
ordinate jurisdiction indicates that the court with whom the petition is first filed, must also first take
cognizance of the settlement of the estate in order to exercise jurisdiction over it to the exclusion of
all other courts.

Conversely, such court, may upon learning that a petition for probate of the decedent's last will has
been presented in another court where the decedent obviously had his conjugal domicile and
resided with his surviving widow and their minor children, and that the allegation of
the intestate petition before it stating that the decedent died intestate may be actually false,
may decline to take cognizance of the petition and hold the petition before it in abeyance, and
instead defer to the second court which has before it the petition for probate of the decedent's
alleged last will.

2. This exactly what the Cebu court did. Upon petitioner-widow's filing with it a motion to dismiss
Lourdes' intestate petition, it issued its order holding in abeyance its action on the dismissal motion
and deferred to the Quezon City court, awaiting its action on the petition for probate before that
court. Implicit in the Cebu court's order was that if the will was duly admitted to probate, by the
Quezon City court, then it would definitely decline to take cognizance of Lourdes' intestate petition
which would thereby be shown to be false and improper, and leave the exercise of jurisdiction to
the Quezon City court, to the exclusion of all other courts. Likewise by its act of deference, the Cebu
court left it to the Quezon City court to resolve the question between the parties whether the
decedent's residence at the time of his death was in Quezon City where he had his conjugal
domicile rather than in Cebu City as claimed by respondents. The Cebu court thus indicated that it
would decline to take cognizance of the intestate petition before it and instead defer to the Quezon
City court, unless the latter would make a negative finding as to the probate petition and the
residence of the decedent within its territory and venue.

3. Under these facts, the Cebu court could not be held to have acted without jurisdiction or with
grave abuse of jurisdiction in declining to take cognizance of the intestate petition and deferring to
the Quezon City court.

Necessarily, neither could the Quezon City court be deemed to have acted without jurisdiction in
taking cognizance of and acting on the probate petition since under Rule 73, section 1, the Cebu
court must first take cognizance over the estate of the decedent and must exercise jurisdiction to
exclude all other courts, which the Cebu court declined to do. Furthermore, as is undisputed, said
rule only lays down a rule of venue and the Quezon City court indisputably had at least equal and
coordinate jurisdiction over the estate.

Since the Quezon City court took cognizance over the probate petition before it and assumed
jurisdiction over the estate, with the consent and deference of the Cebu court, the Quezon City court
should be left now, by the same rule of venue of said Rule 73, to exercise jurisdiction to the
exclusion of all other courts.

Under the facts of the case and where respondents submitted to the Quezon City court
their opposition to probate of the will, but failed to appear at the scheduled hearing despite due
notice, the Quezon City court cannot be declared, as the appellate court did, to have acted without
jurisdiction in admitting to probate the decedent's will and appointing petitioner-widow as executrix
thereof in accordance with the testator's testamentary disposition.
4. The relatively recent case of Uriarte vs. Court of First Instance of Negros Occidental 12 with facts
analogous to the present case 13 is authority against respondent appellate court's questioned decision.

In said case, the Court upheld the doctrine of precedence of probate proceedings over intestate
proceedings in this wise:

It can not be denied that a special proceeding intended to effect the distribution of the
estate of a deceased person, whether in accordance with the law on intestate
succession or in accordance with his will, is a "probate matter" or a proceeding for
the settlement of his estate. It is equally true, however, that in accordance with
settled jurisprudence in this jurisdiction, testate proceedings for the settlement of the
estate of a deceased person take precedence over intestate proceedings for the
same purpose. Thus it has been held repeatedly that, if in the course of intestate
proceedings pending before a court of first instance it is found that the decedent had
left a last will, proceedings for the probate of the latter should replace the intestate
proceedings even if at that state an administrator had already been appointed, the
latter being required to render final account and turn over the estate in his
possession to the executor subsequently appointed. This however, is understood to
be without prejudice that should the alleged last will be rejected or is disapproved,
the proceeding shall continue as an intestacy. As already adverted to, this is a clear
indication that proceedings for the probate of a will enjoy priority over intestate
proceedings. 14

The Court likewise therein upheld the jurisdiction of the second court, (in this case, the Quezon City
court) although opining that certain considerations therein "would seem to support the view that
[therein respondent] should have submitted said will for probate to the Negros Court, [in this case,
the Cebu court] either in a separate special proceeding or in an appropriate motion for said purpose
filed in the already pending Special Proceeding No. 6344," 15 thus:

But the fact is that instead of the aforesaid will being presented for probate to the Negros Court,
Juan Uriarte Zamacona filed the petition for the purpose with the Manila Court. We can not accept
petitioner's contention in this regard that the latter court had no jurisdiction to consider said petition,
albeit we say that it was not the proper venue therefor.

It is well settled in this jurisdiction that wrong venue is merely a waivable procedural
defect, and, in the light of the circumstances obtaining in the instant case, we are of
the opinion, and so hold, that petitioner has waived the right to raise such objection
or is precluded from doing so by laches. It is enough to consider in this connection
that petitioner knew of the existence of a will executed by Juan Uriarte y Goite since
December 19, 1961 when Higinio Uriarte filed his opposition to the initial petition filed
in Special Proceeding No. 6344; that petitioner likewise was served with notice of the
existence (presence) of the alleged last will in the Philippines and of the filing of the
petition for its probate with the Manila Court since August 28, 1962 when Juan
Uriarte Zamacona filed a motion for the dismissal of Special Proceeding No. 6344.
All these notwithstanding, it was only on April 15, 1963 that he filed with the Manila
Court in Special Proceeding No. 51396 an Omnibus motion asking for leave to
intervene and for the dismissal and annulment of all the proceedings had therein up
to that date; thus enabling the Manila Court not only to appoint an administrator with
the will annexed but also to admit said will to probate more than five months earlier,
or more specifically, on October 31, 1962. To allow him now to assail the exercise of
jurisdiction over the probate of the will by the Manila Court and the validity of all the
proceedings had in Special Proceeding No. 51396 would put a premium on his
negligence. Moreover, it must be remembered that this Court is not inclined to annul
proceedings regularly had in a lower court even if the latter was not the proper
venue therefor, if the net result would be to have the same proceedings repeated in
some other court of similar jurisdiction; more so in a case like the present where the
objection against said proceedings is raised too late. 16

5. Under Rule 73, section 1 itself, the Quezon City court's assumption of jurisdiction over the
decedent's estate on the basis of the will duly presented for probate by petitioner-widow and finding
that Quezon City was the firstchoice of residence of the decedent, who had his conjugal home and
domicile therein with the deference in comity duly given by the Cebu court could not be
contested except by appeal from said court in the original case. The last paragraph of said Rule
expressly provides:

... The jurisdiction assumed by a court, so far as it depends on the place of residence
of the decedent, or of the location of his estate, shall not be contested in a suit or
proceeding, except in an appeal from that court, in the original case, or when the
want of jurisdiction appears on the record. (Rule 73)

The exception therein given, viz, "when the want of jurisdiction appears on the record" could
probably be properly invoked, had such deference in comity of the Cebu court to the Quezon City
court not appeared in the record, or had the record otherwise shown that the Cebu court had taken
cognizance of the petition before it and assumed jurisdiction.

6. On the question that Quezon City established to be the residence of the late senator, the
appellate court while recognizing that "the issue is a legitimate one" held in reliance on Borja vs.
Tan 17 that.

... The issue of residence comes within the competence of whichever court is
considered to prevail in the exercise jurisdiction - in this case, the Court of First
Instance of Cebu as held by this Court. Parenthetically, we note that the question of
the residence of the deceased is a serious one, requiring both factual and legal
resolution on the basis of ample evidence to be submitted in the ordinary course of
procedure in the first instance, particularly in view of the fact that the deceased was
better known as the Senator from Cebu and the will purporting to be his also gives
Cebu, besides Quezon City, as his residence. We reiterate that this matter requires
airing in the proper court, as so indicated in the leading and controlling case of Borja
vs. Hon. Bienvenido Tan, et al., G.R. L-7792, July 27, 1955.

In the case at bar, however, the Cebu court declined to take cognizance of the intestate petition first
filed with it and deferred to the testate proceedings filed with the Quezon City court and in effect
asked the Quezon City court to determine the residence of the decedent and whether he did leave a
last will and testament upon which would depend the proper venue of the estate proceedings, Cebu
or Quezon City. The Quezon City court having thus determined in effect for both courts at the
behest and with the deference and consent of the Cebu court thatQuezon City was the actual
residence of the decedent who died testate and therefore the proper venue, the Borja ruling would
seem to have no applicability. It would not serve the practical ends of justice to still require the Cebu
court, if the Borja ruling is to be held applicable and as indicated in the decision under review, to
determine for itself the actual residence of the decedent (when the Quezon City court had already so
determined Quezon City as the actual residence at the Cebu court's behest and respondents have
not seriously questioned this factual finding based on documentary evidence) and if the Cebu court
should likewise determine Quezon City as the actual residence, or its contrary finding reversed on
appeal, only then to allow petitioner-widow after years of waiting and inaction to institute the
corresponding proceedings in Quezon City.

7. With more reason should the Quezon City proceedings be upheld when it is taken into
consideration that Rule 76, section 2 requires that the petition for allowance of a will must show: "(a)
the jurisdictional facts." Such "jurisdictional facts" in probate proceedings, as held by the Court
in Fernando vs. Crisostomo 18 " are the death of the decedent, his residence at the time of his death in
the province where the probate court is sitting, or if he is an inhabitant of a foreign country, his having left
his estate in such province."

This tallies with the established legal concept as restated by Moran that "(T)he probate of a will is a
proceeding in rem. The notice by publication as a pre-requisite to the allowance of a will, is a
constructive notice to the whole world, and when probate is granted, the judgment of the court
is binding upon everybody, even against the State.The probate of a will by a court having jurisdiction
thereof is conclusive as to its due execution and validity." 19 The Quezon City court acted
regularly within its jurisdiction (even if it were to be conceded that Quezon City was not the proper venue
notwithstanding the Cebu court's giving way and deferring to it,) in admitting the decedent's last will to
probate and naming petitioner-widow as executrix thereof. Hence, the Quezon city court's action
should not be set aside by a writ of prohibition for supposed lack of jurisdiction as per the appellate court's
appealed decision, and should instead be sustained in line with Uriarte, supra, where the Court, in
dismissing the certiorari petition challenging the Manila court's action admitting the decedent's will to
probate and distributing the estate in accordance therewith in the second proceeding, held that "it must be
remembered that this Court is not inclined to annul proceedings regularly had in a lower court even if the
latter was not the proper venue therefor, if the net result would be to have the same proceedings
repeated in some other court of similar jurisdiction." As stressed by Chief Justice Moran in Sy Oa, supra,
"the mischievous effect in the administration of justice" of considering the question of residence as
affecting the jurisdiction of the trial court and annulling the whole proceedings only to start all over again
the same proceedings before another court of the same rank in another province "is too obvious to
require comment."

8. If the question of jurisdiction were to be made to depend only on who of the decedent's relatives
gets first to file a petition for settlement of the decedent's estate, then the established jurisprudence
of the Court that Rule 73, section 1 provides only a rule of venue in order to preclude different courts
which may properly assumejurisdiction from doing so and creating conflicts between them to the
detriment of the administration of justice, and that venue is waivable, would be set at naught. As
between relatives who unfortunately do not see eye to eye, it would be converted into a race as to
who can file the petition faster in the court of his/her choice regardless of whether the decedent is
still in cuerpo presente and in disregard of the decedent's actual last domicile, the fact that he left a
last will and testament and the right of his surviving widow named as executrix thereof. Such dire
consequences were certainly not intended by the Rule nor would they be in consonance with public
policy and the orderly administration of justice.

9. It would finally be unjust and inequitable that petitioner-widow, who under all the applicable rules
of venue, and despite the fact that the Cebu court (where respondent Lourdes Cuenco had filed
an intestate petition in the Cebu court earlier by a week's time on 5 March 1964) deferred to the
Quezon City court where petitioner had within fifteen days (on March 12, 1964) after the decedent's
death (on February 25, 1964) timely filed the decedent's last will and petitioned for letters
testamentary and is admittedly entitled to preference in the administration of her husband's
estate, 20 would be compelled under the appealed decision to have to go all the way to Cebu and submit
anew the decedent's will there for probate either in a new proceeding or by asking that the intestate
proceedings be converted into a testate proceeding when under the Rules, the proper venue for
the testate proceedings, as per the facts of record and as already affirmed by the Quezon City court is
Quezon City, where the decedent and petitioner-widow had their conjugal domicile.
It would be an unfair imposition upon petitioner as the one named and entitled to be executrix of the
decedent's last will and settle his estate in accordance therewith, and a disregard of her rights under
the rule on venue and the law on jurisdiction to require her to spend much more time, money and
effort to have to go from Quezon City to the Cebu court everytime she has an important matter of the
estate to take up with the probate court.

It would doubly be an unfair imposition when it is considered that under Rule 73, section 2, 21 since
petitioner's marriage has been dissolved with the death of her husband, their community property and
conjugal estate have to beadministered and liquidated in the estate proceedings of the deceased spouse.
Under the appealed decision, notwithstanding that petitioner resides in Quezon City, and the proper
venue of the testate proceeding was in Quezon City and the Quezon City court properly took cognizance
and exercised exclusive jurisdiction with the deference in comity and consent of the Cebu court, such
proper exercise of jurisdiction would be nullified and petitioner would have to continually leave her
residence in Quezon City and go to Cebu to settle and liquidate even her own community property and
conjugal estate with the decedent.

10. The Court therefore holds under the facts of record that the Cebu court did not act without
jurisdiction nor with grave abuse of discretion in declining to take cognizance of the intestate petition
and instead deferring to thetestate proceedings filed just a week later by petitioner as surviving
widow and designated executrix of the decedent's last will, since the record before it (the petitioner's
opposition and motion to dismiss) showed the falsityof the allegation in the intestate petition that the
decedent had died without a will. It is noteworthy that respondents never challenged by certiorari or
prohibition proceedings the Cebu court's order of 10 April 1964 deferring to the probate proceedings
before the Quezon City court, thus leaving the latter free (pursuant to the Cebu court's order of
deference) to exercise jurisdiction and admit the decedent's will to probate.

For the same reasons, neither could the Quezon City court be held to have acted without jurisdiction
nor with grave abuse of discretion in admitting the decedent's will to probate and appointing
petitioner as executrix in accordance with its testamentary disposition, in the light of the settled
doctrine that the provisions of Rule 73, section 1 lay down only a rule of venue, not of jurisdiction.

Since respondents undisputedly failed to appeal from the Quezon City court's order of May 15, 1964
admitting the will to probate and appointing petitioner as executrix thereof, and said court
concededly has jurisdiction to issue said order, the said order of probate has long since become final
and can not be overturned in a special civic action of prohibition.

11. Finally, it should be noted that in the Supreme Court's exercise of its supervisory authority over
all inferior courts, 22 it may properly determine, as it has done in the case at bar, that venue was properly
assumed by and transferredto the Quezon City court and that it is the interest of justice and in avoidance
of needless delay that the Quezon City court's exercise of jurisdiction over the testate estate of the
decedent (with the due deference and consent of the Cebu court) and its admission to probate of his last
will and testament and appointment of petitioner-widow as administratrix without bond in pursuance of the
decedent's express will and all its orders and actions taken in the testate proceedings before it be
approved and authorized rather than to annul all such proceedings regularly had and to repeat and
duplicate the same proceedings before the Cebu court only to revert once more to the Quezon City court
should the Cebu court find that indeed and in fact, as already determined by the Quezon City court on the
strength of incontrovertible documentary evidence of record, Quezon City was the conjugal residence of
the decedent.

ACCORDINGLY, judgment is hereby rendered reversing the appealed decision and resolution of the
Court of Appeals and the petition for certiorari and prohibition with preliminary injunction originally
filed by respondents with the Court of Appeals (CA-G.R. No. 34104-R) is ordered dismissed. No
costs.
Makalintal, C.J., Zaldivar, Makasiar, Antonio and Esguerra, JJ., concur.

Fernando and Castro, JJ., took no part.

Separate Opinions

BARREDO, J., concurring:

I concur in the main opinion of Mr. Justice Teehankee.

I only want to stress that in my view, the failure of respondents to question within a reasonable time
the laying of the venue in the Quezon City Court of First Instance and the assumption of jurisdiction
by that court, after the Court of First Instance of Cebu deferred in its favor, in order to prevent the
holding therein of any proceeding and trial, and their having filed therein a formal opposition to the
probate of the will, makes them guilty of laches, for which reason they are not entitled to the
equitable relief prayed for in the present petition.

Separate Opinions

BARREDO, J., concurring:

I concur in the main opinion of Mr. Justice Teehankee.

I only want to stress that in my view, the failure of respondents to question within a reasonable time
the laying of the venue in the Quezon City Court of First Instance and the assumption of jurisdiction
by that court, after the Court of First Instance of Cebu deferred in its favor, in order to prevent the
holding therein of any proceeding and trial, and their having filed therein a formal opposition to the
probate of the will, makes them guilty of laches, for which reason they are not entitled to the
equitable relief prayed for in the present petition.

FIRST DIVISION

[G.R. No. 123486. August 12, 1999]

EUGENIA RAMONAL CODOY, and MANUEL RAMONAL, petitioners, vs. EVANGELINE R. CALUGAY,
JOSEPHINE SALCEDO, and EUFEMIA PATIGAS,respondents.

DECISION
PARDO, J.:

Before us is a petition for review on certiorari of the decision of the Court of Appeals[1] and its
resolution denying reconsideration, ruling:

Upon the unrebutted testimony of appellant Evangeline Calugay and witness Matilde Ramonal Binanay,
the authenticity of testators holographic will has been established and the handwriting and signature
therein (exhibit S) are hers, enough to probate said will. Reversal of the judgment appealed from and the
probate of the holographic will in question be called for. The rule is that after plaintiff has completed
presentation of his evidence and the defendant files a motion for judgment on demurrer to evidence on
the ground that upon the facts and the law plaintiff has shown no right to relief, if the motion is granted
and the order to dismissal is reversed on appeal, the movant loses his right to present evidence in his
behalf (Sec. 1 Rule 35 Revised Rules of Court). Judgment may, therefore, be rendered for appellant in
the instant case.

Wherefore, the order appealed from is REVERSED and judgment rendered allowing the probate of the
holographic will of the testator Matilde Seo Vda. de Ramonal.[2]

The facts are as follows:


On April 6, 1990, Evangeline Calugay, Josephine Salcedo and Eufemia Patigas, devisees and legatees
of the holographic will of the deceased Matilde Seo Vda. de Ramonal, filed with the Regional Trial Court,
Misamis Oriental, Branch 18, a petition[3] for probate of the holographic will of the deceased, who died
on January 16, 1990.
In the petition, respondents claimed that the deceased Matilde Seo Vda. de Ramonal, was of sound
and disposing mind when she executed the will on August 30, 1978, that there was no fraud, undue
influence, and duress employed in the person of the testator, and the will was written voluntarily.
The assessed value of the decedents property, including all real and personal property was
about P400,000.00, at the time of her death.[4]
On June 28, 1990, Eugenia Ramonal Codoy and Manuel Ramonal filed an opposition[5] to the
petition for probate, alleging that the holographic will was a forgery and that the same is even
illegible. This gives an impression that a third hand of an interested party other than the true hand of
Matilde Seo Vda. de Ramonal executed the holographic will.
Petitioners argued that the repeated dates incorporated or appearing on the will after every
disposition is out of the ordinary. If the deceased was the one who executed the will, and was not
forced, the dates and the signature should appear at the bottom after the dispositions, as regularly done
and not after every disposition. And assuming that the holographic will is in the handwriting of the
deceased, it was procured by undue and improper pressure and influence on the part of the
beneficiaries, or through fraud and trickery.
Respondents presented six (6) witnesses and various documentary evidence. Petitioners instead of
presenting their evidence, filed a demurrer[6] to evidence, claiming that respondents failed to establish
sufficient factual and legal basis for the probate of the holographic will of the deceased Matilde Seo Vda.
de Ramonal.
On November 26, 1990, the lower Court issued an order, the dispositive portion of which reads:
WHEREFORE, in view of the foregoing consideration, the Demurrer to Evidence having being well taken,
same is granted, and the petition for probate of the document (Exhibit S) on the purported Holographic
Will of the late Matilde Seo Vda. de Ramonal, is denied for insufficiency of evidence and lack of merits.[7]

On December 12, 1990, respondents filed a notice of appeal,[8] and in support of their appeal, the
respondents once again reiterated the testimony of the following witnesses, namely: (1) Augusto Neri;
(2) Generosa Senon; (3) Matilde Ramonal Binanay; (4) Teresita Vedad; (5) Fiscal Rodolfo Waga; and (6)
Evangeline Calugay.
To have a clear understanding of the testimonies of the witnesses, we recite an account of their
testimonies.
Augusto Neri, Clerk of Court, Court of First Instance of Misamis Oriental, where the special
proceedings for the probate of the holographic will of the deceased was filed. He produced and
identified the. records of the case. The documents presented bear the signature of the deceased,
Matilde Seo Vda. de Ramonal, for the purpose of laying the basis for comparison of the handwriting of
the testatrix, with the writing treated or admitted as genuine by the party against whom the evidence is
offered.
Generosa Senon, election registrar of Cagayan de Oro, was presented to produce and identify the
voters affidavit of the decedent. However, the voters affidavit was not produced for the same was
already destroyed and no longer available.
Matilde Ramonal Binanay, testified that the deceased Matilde Seo Vda. de Ramonal was her aunt,
and that after the death of Matildes husband, the latter lived with her in her parents house for eleven
(11) years, from 1958 to 1969. During those eleven (11) years of close association with the deceased,
she acquired familiarity with her signature and handwriting as she used to accompany her (deceased
Matilde Seo Vda. de Ramonal) in collecting rentals from her various tenants of commercial buildings,
and the deceased always issued receipts. In addition to this, she (witness Matilde Binanay) assisted the
deceased in posting the records of the accounts, and carried personal letters of the deceased to her
creditors.
Matilde Ramonal Binanay further testified that at the time of the death of Matilde Vda. de
Ramonal, she left a holographic will dated August 30, 1978, which was personally and entirely written,
dated and signed, by the deceased and that all the dispositions therein, the dates, and the signatures in
said will, were that of the deceased.
Fiscal Rodolfo Waga testified that before he was appointed City Fiscal of Cagayan de Oro, he was a
practicing lawyer, and handled all the pleadings and documents signed by the deceased in connection
with the intestate proceedings of her late husband, as a result of which he is familiar with the
handwriting of the latter. He testified that the signature appearing in the holographic will was similar to
that of the deceased, Matilde Seo Vda. de Ramonal, but he can not be sure.
The fifth witness presented was Mrs. Teresita Vedad, an employee of the Department of
Environment and Natural Resources, Region 10. She testified that she processed the application of the
deceased for pasture permit and was familiar with the signature of the deceased, since the deceased
signed documents in her presence, when the latter was applying for pasture permit.
Finally, Evangeline Calugay, one of the respondents, testified that she had lived with the deceased
since birth, and was in fact adopted by the latter. That after a long period of time she became familiar
with the signature of the deceased. She testified that the signature appearing in the holographic will is
the true and genuine signature of Matilde Seo Vda. de Ramonal.
The holographic will which was written in Visayan, is translated in English as follows:

Instruction

August 30, 1978

1. My share at Cogon, Raminal Street, for Evangeline Calugay.

(Sgd) Matilde Vda de Ramonal

August 30, 1978

2. Josefina Salcedo must be given 1,500 square meters at Pinikitan Street.

(Sgd) Matilde Vda de Ramonal

August 30, 1978

3. My jewelrys shall be divided among:

1. Eufemia Patigas

2. Josefina Salcedo

3. Evangeline Calugay

(Sgd)Matilde Vda de Ramonal

August 30, 1978

4. I bequeath my one (1) hectare land at Mandumol, Indahag to Evangeline R. Calugay

(Sgd) Matilde Vda de Ramonal

"August 30, 1978

5. Give the 2,500 Square Meters at Sta. Cruz Ramonal Village in favor of Evangeline R. Calugay, Helen
must continue with the Sta. Cruz, once I am no longer around.

(Sgd) Matilde Vda de Ramonal

August 30, 1978

6. Bury me where my husband Justo is ever buried.


(Sgd) Matilde Vda de Ramonal

"August 30,1978

Gene and Manuel:

"Follow my instruction in order that I will rest peacefully.

Mama

Matilde Vda de Ramonal

On October 9, 1995, the Court of Appeals, rendered decision[9] ruling that the appeal was
meritorious. Citing the decision in the case of Azaola vs. Singson, 109 Phil. 102, penned by Mr. Justice J.
B. L. Reyes, a recognized authority in civil law, the Court of Appeals held:

x x x even if the genuineness of the holographic will were contested, we are of the opinion that Article
811 of our present civil code can not be interpreted as to require the compulsory presentation of three
witnesses to identify the handwriting of the testator, under penalty of having the probate denied. Since
no witness may have been present at the execution of the holographic will, none being required by law
(art. 810, new civil code), it becomes obvious that the existence of witnesses possessing the requisite
qualifications is a matter beyond the control of the proponent. For it is not merely a question of finding
and producing any three witnesses; they must be witnesses who know the handwriting and signature of
the testator and who can declare (truthfully, of course, even if the law does not express) that the will
and the signature are in the handwriting of the testator. There may be no available witness acquainted
with the testators hand; or even if so familiarized, the witness may be unwilling to give a positive
opinion.Compliance with the rule of paragraph 1 of article 811 may thus become an impossibility. That is
evidently the reason why the second paragraph of article 811 prescribes that

in the absence of any competent witness referred to in the preceding paragraph, and if the court deems
it necessary, expert testimony may be resorted to.

As can be seen, the law foresees the possibility that no qualified witness may be found (or what
amounts to the same thing, that no competent witness may be willing to testify to the authenticity of
the will), and provides for resort to expert evidence to supply the deficiency.

It may be true that the rule of this article (requiring that three witnesses be presented if the will is
contested and only one if no contest is had) was derived from the rule established for ordinary
testaments (CF Cabang vs. Delfinado, 45 PHIL 291; Tolentino v. Francisco, 57 PHIL 742). But it can not be
ignored that the requirement can be considered mandatory only in case of ordinary testaments,
precisely because the presence of at least three witnesses at the execution of ordinary wills is made by
law essential to their validity (Art. 805). Where the will is holographic, no witness need be present
(art.10), and the rule requiring production of three witnesses must be deemed merely permissive if
absurd results are to be avoided.

Again, under Art.811, the resort to expert evidence is conditioned by the words if the court deem it
necessary, which reveal that what the law deems essential is that the court should be convinced of the
wills authenticity. Where the prescribed number of witnesses is produced and the court is convinced by
their testimony that the will is genuine, it may consider it unnecessary to call for expert evidence. On the
other hand, if no competent witness is available, or none of those produced is convincing, the court may
still, and in fact it should resort to handwriting experts. The duty of the court, in fine, is to exhaust all
available lines of inquiry, for the state is as much interested as the proponent that the true intention of
the testator be carried into effect.

Paraphrasing Azaola vs. Singson, even if the genuineness of the holographic will were contested, Article
811 of the civil code cannot be interpreted as to require the compulsory presentation of three witnesses
to identify the handwriting of the testator, under penalty of the having the probate denied. No witness
need be present in the execution of the holographic will. And the rule requiring the production of three
witnesses is merely permissive. What the law deems essential is that the court is convinced of the
authenticity of the will. Its duty is to exhaust all available lines of inquiry, for the state is as much
interested in the proponent that the true intention of the testator be carried into effect. And because
the law leaves it to the trial court to decide if experts are still needed, no unfavorable inference can be
drawn from a partys failure to offer expert evidence, until and unless the court expresses dissatisfaction
with the testimony of the lay witnesses.[10]

According to the Court of Appeals, Evangeline Calugay, Matilde Ramonal Binanay and other
witnesses definitely and in no uncertain terms testified that the handwriting and signature in the
holographic will were those of the testator herself.
Thus, upon the unrebutted testimony of appellant Evangeline Calugay and witness Matilde
Ramonal Binanay, the Court of Appeals sustained the authenticity of the holographic will and the
handwriting and signature therein, and allowed the will to probate.
Hence, this petition.
The petitioners raise the following issues:
(1) Whether or not the ruling of the case of Azaola vs. Singson, 109 Phil. 102, relied upon by
the respondent Court of Appeals, was applicable to the case.
(2) Whether or not the Court of Appeals erred in holding that private respondents had been
able to present credible evidence to prove that the date, text, and signature on the
holographic will were written entirely in the hand of the testatrix.
(3) Whether or not the Court of Appeals erred in not analyzing the signatures in the
holographic will of Matilde Seo Vda. de Ramonal.
In this petition, the petitioners ask whether the provisions of Article 811 of the Civil Code are
permissive or mandatory. The article provides, as a requirement for the probate of a contested
holographic will, that at least three witnesses explicitly declare that the signature in the will is the
genuine signature of the testator.
We are convinced, based on the language used, that Article 811 of the Civil Code is mandatory. The
word shall connotes a mandatory order. We have ruled that shall in a statute commonly denotes an
imperative obligation and is inconsistent with the idea of discretion and that the presumption is that the
word shall, when used in a statute is mandatory.[11]
Laws are enacted to achieve a goal intended and to guide against an evil or mischief that aims to
prevent. In the case at bar, the goal to achieve is to give effect to the wishes of the deceased and the
evil to be prevented is the possibility that unscrupulous individuals who for their benefit will employ
means to defeat the wishes of the testator.
So, we believe that the paramount consideration in the present petition is to determine the true
intent of the deceased. An exhaustive and objective consideration of the evidence is imperative to
establish the true intent of the testator.
It will be noted that not all the witnesses presented by the respondents testified explicitly that they
were familiar with the handwriting of the testator. In the case of Augusto Neri, clerk of court, Court of
First Instance, Misamis Oriental, he merely identified the record of Special Proceedings No. 427 before
said court. He was not presented to declare explicitly that the signature appearing in the holographic
was that of the deceased.
Generosa E. Senon, the election registrar of Cagayan de Oro City, was presented to identify the
signature of the deceased in the voters affidavit, which was not even produced as it was no longer
available.
Matilde Ramonal Binanay, on the other hand, testified that:
Q. And you said for eleven (11) years Matilde Vda de Ramonal resided with your parents at Pinikitan,
Cagayan de Oro City. Would you tell the court what was your occupation or how did Matilde
Vda de Ramonal keep herself busy that time?
A. Collecting rentals.
Q. From where?
A. From the land rentals and commercial buildings at Pabayo-Gomez streets.[12]
xxx
Q. Who sometime accompany her?
A. I sometimes accompany her
Q. In collecting rentals does she issue receipts?
A. Yes, sir.[13]
xxx
Q. Showing to you the receipt dated 23 October 1979, is this the one you are referring to as one of
the receipts which she issued to them?
A. Yes, sir.
Q. Now there is that signature of Matilde vda. De Ramonal, whose signature is that Mrs. Binanay?
A. Matilde vda. De Ramonal.
Q. Why do you say that that is a signature of Matilde vda. De Ramonal?
A. I am familiar with her signature.
Q. Now, you tell the court Mrs. Binanay, whether you know Matilde vda de Ramonal kept records of
the accounts of her tenants?
A. Yes, sir.
Q. Why do you say so?
A. Because we sometimes post a record of accounts in behalf of Matilde Vda. De Ramonal.
Q. How is this record of accounts made? How is this reflected?
A. In handwritten.[14]
xxx
Q. In addition to collection of rentals, posting records of accounts of tenants and deed of sale which
you said what else did you do to acquire familiarity of the signature of Matilde Vda De Ramonal?
A. Posting records.
Q. Aside from that?
A. Carrying letters.
Q. Letters of whom?
A. Matilde
Q. To whom?
A. To her creditors.[15]
xxx
Q. You testified that at the time of her death she left a will. I am showing to you a document with its
title tugon is this the document you are referring to?
A. Yes, sir.
Q. Showing to you this exhibit S, there is that handwritten tugon, whose handwriting is this?
A. My aunt.
Q. Why do you say this is the handwriting of your aunt?
A. Because I am familiar with her signature.[16]
What Ms. Binanay saw were pre-prepared receipts and letters of the deceased, which she either
mailed or gave to her tenants. She did not declare that she saw the deceased sign a document or write a
note.
Further, during the cross-examination, the counsel for petitioners elicited the fact that the will was
not found in the personal belongings of the deceased but was in the possession of Ms. Binanay. She
testified that:
Q. Mrs. Binanay, when you were asked by counsel for the petitioners if the late Matilde Seno vda de
Ramonal left a will you said, yes?
A. Yes, sir.
Q. Who was in possession of that will?
A. I.
Q. Since when did you have the possession of the will?
A. It was in my mothers possession.
Q. So, it was not in your possession?
A. Sorry, yes.
Q. And when did you come into possession since as you said this was originally in the possession of
your mother?
A. 1985.[17]
xxx
Q. Now, Mrs. Binanay was there any particular reason why your mother left that will to you and
therefore you have that in your possession?
A. It was not given to me by my mother, I took that in the aparador when she died.
Q. After taking that document you kept it with you?
A. I presented it to the fiscal.
Q. For what purpose?
A. Just to seek advice.
Q. Advice of what?
A. About the will.[18]
In her testimony it was also evident that Ms. Binanay kept the fact about the will from petitioners,
the legally adopted children of the deceased. Such actions put in issue her motive of keeping the will a
secret to petitioners and revealing it only after the death of Matilde Seo Vda. de Ramonal.
In the testimony of Ms. Binanay, the following were established:
Q. Now, in 1978 Matilde Seno Vda de Ramonal was not yet a sickly person is that correct?
A. Yes, sir.
Q. She was up and about and was still uprightly and she could walk agilely and she could go to her
building to collect rentals, is that correct?
A. Yes, sir.[19]
xxx
Q. Now, let us go to the third signature of Matilde Ramonal. Do you know that there are retracings in
the word Vda.?
A. Yes, a little. The letter L is continuous.
Q. And also in Matilde the letter L is continued to letter D?
A. Yes, sir.
Q. Again the third signature of Matilde Vda de Ramonal the letter L in Matilde is continued towards
letter D.
A. Yes, sir.
Q. And there is a retracing in the word Vda.?
A. Yes, sir.[20]
xxx
Q. Now, that was 1979, remember one year after the alleged holographic will. Now, you identified a
document marked as Exhibit R. This is dated January 8,1978 which is only about eight months
from August 30,1978. Do you notice that the signature Matilde Vda de Ramonal is beautifully
written and legible?
A. Yes, sir the handwriting shows that she was very exhausted.
Q. You just say that she was very exhausted while that in 1978 she was healthy was not sickly and she
was agile. Now, you said she was exhausted?
A. In writing.
Q. How did you know that she was exhausted when you were not present and you just tried to
explain yourself out because of the apparent inconsistencies?
A. That was I think. (sic)
Q. Now, you already observed this signature dated 1978, the same year as the alleged holographic
will. In exhibit I, you will notice that there is no retracing; there is no hesitancy and the signature
was written on a fluid movement. x x x And in fact , the name Eufemia R. Patigas here refers to
one of the petitioners?
A. Yes, sir.
Q. You will also notice Mrs. Binanay that it is not only with the questioned signature appearing in the
alleged holographic will marked as Exhibit X but in the handwriting themselves, here you will
notice the hesitancy and tremors, do you notice that?
A. Yes, sir.[21]
Evangeline Calugay declared that the holographic will was written, dated and signed in the
handwriting of the testator. She testified that:
Q. You testified that you stayed with the house of the spouses Matilde and Justo Ramonal for the
period of 22 years. Could you tell the court the services if any which you rendered to Matilde
Ramonal?
A. During my stay I used to go with her to the church, to the market and then to her transactions.
Q. What else? What services that you rendered?
A. After my college days I assisted her in going to the bank, paying taxes and to her lawyer.
Q. What was your purpose of going to her lawyer?
A. I used to be her personal driver.
Q. In the course of your stay for 22 years did you acquire familiarity of the handwriting of Matilde
Vda de Ramonal?
A. Yes, sir.
Q. How come that you acquired familiarity?
A. Because I lived with her since birth.[22]
xxx
Q. Now, I am showing to you Exhibit S which is captioned tugon dated Agosto 30, 1978 there is a
signature here below item No. 1, will you tell this court whose signature is this?
A. Yes, sir, that is her signature.
Q. Why do you say that is her signature?
A. I am familiar with her signature.[23]
So, the only reason that Evangeline can give as to why she was familiar with the handwriting of the
deceased was because she lived with her since birth. She never declared that she saw the deceased
write a note or sign a document.
The former lawyer of the deceased, Fiscal Waga, testified that:
Q. Do you know Matilde Vda de Ramonal?
A. Yes, sir I know her because she is my godmother the husband is my godfather. Actually I am
related to the husband by consanguinity.
Q. Can you tell the name of the husband?
A. The late husband is Justo Ramonal.[24]
xxx
Q. Can you tell this court whether the spouses Justo Ramonal and Matilde Ramonal have legitimate
children?
A. As far as I know they have no legitimate children.[25]
xxx
Q. You said after becoming a lawyer you practice your profession? Where?
A. Here in Cagayan de Oro City.
Q. Do you have services rendered with the deceased Matilde vda de Ramonal?
A. I assisted her in terminating the partition, of properties.
Q. When you said assisted, you acted as her counsel? Any sort of counsel as in what case is that,
Fiscal?
A. It is about the project partition to terminate the property, which was under the court before.[26]
xxx
Q. Appearing in special proceeding no. 427 is the amended inventory which is marked as exhibit N of
the estate of Justo Ramonal and there appears a signature over the type written word Matilde
vda de Ramonal, whose signature is this?
A. That is the signature of Matilde Vda de Ramonal.
Q. Also in exhibit n-3, whose signature is this?
A. This one here that is the signature of Mrs. Matilde vda de Ramonal.[27]
xxx
Q. Aside from attending as counsel in that Special Proceeding Case No. 427 what were the other
assistance wherein you were rendering professional service to the deceased Matilde Vda de
Ramonal?
A. I can not remember if I have assisted her in other matters but if there are documents to show that
I have assisted then I can recall.[28]
xxx
Q. Now, I am showing to you exhibit S which is titled tugon, kindly go over this document, Fiscal
Waga and tell the court whether you are familiar with the handwriting contained in that
document marked as exhibit S?
A. I am not familiar with the handwriting.
Q. This one, Matilde Vda de Ramonal, whose signature is this?
A. I think this signature here it seems to be the signature of Mrs. Matilde vda de Ramonal.
Q. Now, in item No. 2 there is that signature here of Matilde Vda de Ramonal, can you tell the court
whose signature is this?
A. Well, that is similar to that signature appearing in the project of partition.
Q. Also in item no. 3 there is that signature Matilde Vda de Ramonal, can you tell the court whose
signature is that?
A. As I said, this signature also seems to be the signature of Matilde vda de Ramonal.
Q. Why do you say that?
A. Because there is a similarity in the way it is being written.
Q. How about this signature in item no. 4, can you tell the court whose signature is this?
A. The same is true with the signature in item no. 4. It seems that they are similar.[29]
xxx
Q. Mr. Prosecutor, I heard you when you said that the signature of Matilde Vda de Ramonal
Appearing in exhibit S seems to be the signature of Matilde vda de Ramonal?
A. Yes, it is similar to the project of partition.
Q. So you are not definite that this is the signature of Matilde vda de Ramonal. You are merely
supposing that it seems to be her signature because it is similar to the signature of the project
of partition which you have made?
A. That is true.[30]
From the testimonies of these witnesses, the Court of Appeals allowed the will to probate and
disregard the requirement of three witnesses in case of contested holographic will, citing the decision in
Azaola vs. Singson,[31] ruling that the requirement is merely directory and not mandatory.
In the case of Ajero vs. Court of Appeals,[32] we said that the object of the solemnities surrounding
the execution of wills is to close the door against bad faith and fraud, to avoid substitution of wills and
testaments and to guaranty their truth and authenticity. Therefore, the laws on this subject should be
interpreted in such a way as to attain these primordial ends. But, on the other hand, also one must not
lose sight of the fact that it is not the object of the law to restrain and curtail the exercise of the right to
make a will.
However, we cannot eliminate the possibility of a false document being adjudged as the will of the
testator, which is why if the holographic will is contested, that law requires three witnesses to declare
that the will was in the handwriting of the deceased.
The will was found not in the personal belongings of the deceased but with one of the respondents,
who kept it even before the death of the deceased. In the testimony of Ms. Binanay, she revealed that
the will was in her possession as early as 1985, or five years before the death of the deceased.
There was no opportunity for an expert to compare the signature and the handwriting of the
deceased with other documents signed and executed by her during her lifetime. The only chance at
comparison was during the cross-examination of Ms. Binanay when the lawyer of petitioners asked Ms.
Binanay to compare the documents which contained the signature of the deceased with that of the
holographic will and she is not a handwriting expert. Even the former lawyer of the deceased expressed
doubts as to the authenticity of the signature in the holographic will.
A visual examination of the holographic will convince us that the strokes are different when
compared with other documents written by the testator. The signature of the testator in some of the
disposition is not readable. There were uneven strokes, retracing and erasures on the will.
Comparing the signature in the holographic will dated August 30, 1978,[33] and the signatures in
several documents such as the application letter for pasture permit dated December 30, 1980,[34] and a
letter dated June 16, 1978,[35] the strokes are different. In the letters, there are continuous flows of the
strokes, evidencing that there is no hesitation in writing unlike that of the holographic will. We,
therefore, cannot be certain that the holographic will was in the handwriting by the deceased.
IN VIEW WHEREOF, the decision appealed from is SET ASIDE. The records are ordered remanded to
the court of origin with instructions to allow petitioners to adduce evidence in support of their
opposition to the probate of the holographic will of the deceased Matilde Seo Vda. de Ramonal.
No costs.
SO ORDERED.
Davide Jr., C.J., (Chairman), Puno, Kapunan, and Ynares-Santiago, JJ., concur.

G.R. No. L-55509 April 27, 1984

ETHEL GRIMM ROBERTS, petitioner,


vs.
JUDGE TOMAS R. LEONIDAS, Branch 38, Court of First Instance of Manila; MAXINE TATE-
GRIMM, EDWARD MILLER GRIMM II and LINDA GRIMM, respondents.

N. J. Quisumbing and Associates for petitioners.

Angara, Abello, Concepcion, Regala and Cruz for respondents.

AQUINO, J.: + . wph!1

The question in this case is whether a petition for allowance of wills and to annul a partition,
approved in anintestate proceeding by Branch 20 of the Manila Court of First Instance, can be
entertained by its Branch 38 (after a probate in the Utah district court).

Antecedents. Edward M. Grimm an American resident of Manila, died at 78 in the Makati Medical
Center on November 27, 1977. He was survived by his second wife, Maxine Tate Grimm and their
two children, named Edward Miller Grimm II (Pete) and Linda Grimm and by Juanita Grimm Morris
and Ethel Grimm Roberts (McFadden), his two children by a first marriage which ended in divorce
(Sub-Annexes A and B. pp. 36-47, Rollo).

He executed on January 23, 1959 two wills in San Francisco, California. One will disposed of his
Philippine estate which he described as conjugal property of himself and his second wife. The
second win disposed of his estate outside the Philippines.

In both wills, the second wife and two children were favored. The two children of the first
marriage were given their legitimes in the will disposing of the estate situated in this country. In the
will dealing with his property outside this country, the testator said: t.hqw

I purposely have made no provision in this will for my daughter, Juanita Grimm
Morris, or my daughter, Elsa Grimm McFadden (Ethel Grimm Roberts), because I
have provided for each of them in a separate will disposing of my Philippine property.
(First clause, pp. 43-47, Rollo).

The two wills and a codicil were presented for probate by Maxine Tate Grimm and E. LaVar Tate on
March 7, 1978 in Probate No. 3720 of the Third Judicial District Court of Tooele County, Utah.
Juanita Grimm Morris of Cupertino, California and Mrs. Roberts of 15 C. Benitez Street, Horseshoe
Village, Quezon City were notified of the probate proceeding (Sub-Annex C, pp. 48-55, Rollo).

Maxine admitted that she received notice of the intestate petition filed in Manila by Ethel in January,
1978 (p. 53, Rollo). In its order dated April 10, 1978, the Third Judicial District Court admitted to
probate the two wills and the codicil It was issued upon consideration of the stipulation dated April 4,
1978 "by and between the attorneys for Maxine Tate Grimm, Linda Grimm, Edward Miller Grimm II,
E. LaVar Tate, Juanita Kegley Grimm (first wife), Juanita Grimm Morris and Ethel Grimm Roberts"
(Annex C, pp. 48-51, Rollo).

Two weeks later, or on April 25, 1978, Maxine and her two children Linda and Pete, as the first
parties, and Ethel, Juanita Grimm Morris and their mother Juanita Kegley Grimm as the second
parties, with knowledge of the intestate proceeding in Manila, entered into a compromise
agreement in Utah regarding the estate. It was signed by David E. Salisbury and Donald B.
Holbrook, as lawyers of the parties, by Pete and Linda and the attorney-in-fact of Maxine and by the
attorney-in-fact of Ethel, Juanita Grimm Morris and Juanita Kegley Grimm.

In that agreement, it was stipulated that Maxine, Pete and Ethel would be designated as personal
representatives (administrators) of Grimm's Philippine estate (par. 2). It was also stipulated that
Maxine's one-half conjugal share in the estate should be reserved for her and that would not be less
than $1,500,000 plus the homes in Utah and Santa Mesa, Manila (par. 4). The agreement indicated
the computation of the "net distributable estate". It recognized that the estate was liable to pay the
fees of the Angara law firm (par. 5).

It was stipulated in paragraph 6 that the decedent's four children "shall share equally in the Net
Distributable Estate" and that Ethel and Juanita Morris should each receive at least 12-1/2% of the
total of the net distributable estate and marital share. A supplemental memorandum also dated April
25, 1978 was executed by the parties (Sub-Annex F, pp. 49-61, Annex, F-1, pp. 75-76, Testate
case).

Intestate proceeding No. 113024.-At this juncture, it should be stated that forty- three days after
Grimm's death, or January 9, 1978, his daughter of the first marriage, Ethel, 49, through lawyers
Deogracias T. Reyes and. Gerardo B. Macaraeg, filed with Branch 20 of the Manila Court of First
Instance intestate proceeding No. 113024for the settlement of his estate. She was named special
administratrix.

On March 11, the second wife, Maxine, through the Angara law office, filed an opposition and motion
to dismiss the intestate proceeding on the ground of the pendency of Utah of a proceeding for the
probate of Grimm's will. She also moved that she be appointed special administratrix, She submitted
to the court a copy of Grimm's will disposing of his Philippine estate. It is found in pages 58 to 64 of
the record.

The intestate court in its orders of May 23 and June 2 noted that Maxine, through a new lawyer,
William C. Limqueco (partner of Gerardo B. Macaraeg, p. 78, testate case withdrew that opposition
and motion to dismiss and, at the behest of Maxine, Ethel and Pete, appointed them joint
administrators. Apparently, this was done pursuant to the aforementioned Utah compromise
agreement. The court ignored the will already found in the record.

The three administrators submitted an inventory. With the authority and approval of the court, they
sold for P75,000 on March 21, 1979 the so-called Palawan Pearl Project, a business owned by the
deceased. Linda and Juanita allegedly conformed with the sale (pp. 120-129, Record). It turned out
that the buyer, Makiling Management Co., Inc., was incorporated by Ethel and her husband, Rex
Roberts, and by lawyer Limqueco (Annex L, p. 90, testate case).

Also with the court's approval and the consent of Linda and Juanita, they sold for P1,546,136 to
Joseph Server and others 193,267 shares of RFM Corporation (p. 135, Record).

Acting on the declaration of heirs and project of partition signed and filed by lawyers Limqueco and
Macaraeg (not signed by Maxine and her two children), Judge Conrado M. Molina in his order of July
27, 1979 adjudicated to Maxine onehalf (4/8) of the decedent's Philippine estate and one-eighth (1/8)
each to his four children or 12-1/2% (pp. 140-142, Record). No mention at all was made of the will in
that order.

Six days later, or on August 2, Maxine and her two children replaced Limqueco with Octavio del
Callar as their lawyer who on August 9, moved to defer approval of the project of partition. The court
considered the motion moot considering that it had already approved the declaration of heirs and
project of partition (p. 149, Record).

Lawyer Limqueco in a letter to Maxine dated August 2, 1979 alleged that he was no longer
connected with Makiling Management Co., Inc. when the Palawan Pearl Project was sold: that it was
Maxine's son Pete who negotiated the sale with Rex Roberts and that he (Limqueco) was going to
sue Maxine for the lies she imputed to him (Annex H, p. 78, testate case).

Ethel submitted to the court a certification of the Assistant Commissioner of Internal Revenue dated
October 2, 1979. It was stated therein that Maxine paid P1,992,233.69 as estate tax and penalties
and that he interposed no objection to the transfer of the estate to Grimm's heirs (p. 153, Record).
The court noted the certification as in conformity with its order of July 27, 1979.

After November, 1979 or for a period of more than five months, there was no movement or activity in
the intestate case. On April 18, 1980 Juanita Grimm Morris, through Ethel's lawyers, filed a motion
for accounting "so that the Estate properties can be partitioned among the heirs and the present
intestate estate be closed." Del Callar, Maxine's lawyer was notified of that motion.

Before that motion could be heard, or on June 10, 1980, the Angara law firm filed again its
appearance in collaboration with Del Callar as counsel for Maxine and her two children, Linda and
Pete. It should be recalled that the firm had previously appeared in the case as Maxine's counsel on
March 11, 1978, when it filed a motion to dismiss the intestate proceeding and furnished the court
with a copy of Grimm's will. As already noted, the firm was then superseded by lawyer Limqueco.

Petition to annul partition and testate proceeding No. 134559. On September 8, 1980, Rogelio A.
Vinluan of the Angara law firm in behalf of Maxine, Pete and Linda, filed in Branch 38 of the lower
court a petition praying for the probate of Grimm's two wills (already probated in Utah), that the 1979
partition approved by the intestate court be set aside and the letters of administration revoked, that
Maxine be appointed executrix and that Ethel and Juanita Morris be ordered to account for the
properties received by them and to return the same to Maxine (pp. 25-35, Rollo).

Grimm's second wife and two children alleged that they were defraud due to the machinations of the
Roberts spouses, that the 1978 Utah compromise agreement was illegal, that the intestate
proceeding is void because Grimm died testate and that the partition was contrary to the decedent's
wills.

Ethel filed a motion to dismiss the petition. Judge Leonidas denied it for lack of merit in his order of
October 27, 1980. Ethel then filed a petition for certiorari and prohibition in this Court, praying that
the testate proceeding be dismissed, or. alternatively that the two proceedings be consolidated and
heard in Branch 20 and that the matter of the annulment of the Utah compromise agreement be
heard prior to the petition for probate (pp. 22-23, Rollo).

Ruling. We hold that respondent judge did not commit any grave abuse of discretion, amounting
to lack of jurisdiction, in denying Ethel's motion to dismiss.

A testate proceeding is proper in this case because Grimm died with two wills and "no will shall pass
either real or personal property unless it is proved and allowed" (Art. 838, Civil Code; sec. 1, Rule
75, Rules of Court).

The probate of the will is mandatory (Guevara vs. Guevara, 74 Phil. 479 and 98 Phil. 249; Baluyot
vs. Panio, L-42088, May 7, 1976, 71 SCRA 86). It is anomalous that the estate of a person who
died testate should be settled in an intestate proceeding. Therefore, the intestate case should be
consolidated with the testate proceeding and the judge assigned to the testate proceeding should
continue hearing the two cases.

Ethel may file within twenty days from notice of the finality of this judgment an opposition and answer
to the petition unless she considers her motion to dismiss and other pleadings sufficient for the
purpose. Juanita G. Morris, who appeared in the intestate case, should be served with copies of
orders, notices and other papers in the testate case.

WHEREFORE the petition is dismissed. The temporary restraining order is dissolved. No costs.

SO ORDERED. 1w ph1.t

Makasiar (Chairman), Guerrero and De Castro, JJ., concur.

Escolin, J., concur in the result.

Concepcion, Jr. and Abad Santos, JJ., took no part.

G.R. No. L-62952 October 9, 1985

SOFIA J. NEPOMUCENO, petitioner,


vs.
THE HONORABLE COURT OF APPEALS, RUFINA GOMEZ, OSCAR JUGO ANG, CARMELITA
JUGO,respondents.

GUTIERREZ, JR., J.:

This is a petition for certiorari to set aside that portion of the decision of the respondent Court of
Appeals (now intermediate Appellate Court) dated June 3, 1982, as amended by the resolution
dated August 10, 1982, declaring as null and void the devise in favor of the petitioner and the
resolution dated December 28, 1982 denying petitioner's motion for reconsideration.

Martin Jugo died on July 16, 1974 in Malabon, Rizal. He left a last Will and Testament duly signed
by him at the end of the Will on page three and on the left margin of pages 1, 2 and 4 thereof in the
presence of Celestina Alejandro, Myrna C. Cortez, and Leandro Leano, who in turn, affixed their
signatures below the attestation clause and on the left margin of pages 1, 2 and 4 of the Will in the
presence of the testator and of each other and the Notary Public. The Will was acknowledged before
the Notary Public Romeo Escareal by the testator and his three attesting witnesses.

In the said Will, the testator named and appointed herein petitioner Sofia J. Nepomuceno as his sole
and only executor of his estate. It is clearly stated in the Will that the testator was legally married to a
certain Rufina Gomez by whom he had two legitimate children, Oscar and Carmelita, but since 1952,
he had been estranged from his lawfully wedded wife and had been living with petitioner as husband
and wife. In fact, on December 5, 1952, the testator Martin Jugo and the petitioner herein, Sofia J.
Nepomuceno were married in Victoria, Tarlac before the Justice of the Peace. The testator devised
to his forced heirs, namely, his legal wife Rufina Gomez and his children Oscar and Carmelita his
entire estate and the free portion thereof to herein petitioner. The Will reads in part:
Art. III. That I have the following legal heirs, namely: my aforementioned legal wife,
Rufina Gomez, and our son, Oscar, and daughter Carmelita, both surnamed Jugo,
whom I declare and admit to be legally and properly entitled to inherit from me; that
while I have been estranged from my above-named wife for so many years, I cannot
deny that I was legally married to her or that we have been separated up to the
present for reasons and justifications known fully well by them:

Art. IV. That since 1952, 1 have been living, as man and wife with one Sofia J.
Nepomuceno, whom I declare and avow to be entitled to my love and affection, for all
the things which she has done for me, now and in the past; that while Sofia J.
Nepomuceno has with my full knowledge and consent, did comport and represent
myself as her own husband, in truth and in fact, as well as in the eyes of the law, I
could not bind her to me in the holy bonds of matrimony because of my
aforementioned previous marriage;

On August 21, 1974, the petitioner filed a petition for the probate of the last Will and Testament of
the deceased Martin Jugo in the Court of First Instance of Rizal, Branch XXXIV, Caloocan City and
asked for the issuance to her of letters testamentary.

On May 13, 1975, the legal wife of the testator, Rufina Gomez and her children filed an opposition
alleging inter alia that the execution of the Will was procured by undue and improper influence on the
part of the petitioner; that at the time of the execution of the Will, the testator was already very sick
and that petitioner having admitted her living in concubinage with the testator, she is wanting in
integrity and thus, letters testamentary should not be issued to her.

On January 6, 1976, the lower court denied the probate of the Will on the ground that as the testator
admitted in his Will to cohabiting with the petitioner from December 1952 until his death on July 16,
1974, the Will's admission to probate will be an Idle exercise because on the face of the Will, the
invalidity of its intrinsic provisions is evident.

The petitioner appealed to the respondent-appellate court.

On June 2, 1982, the respondent court set aside the decision of the Court of First Instance of Rizal
denying the probate of the will. The respondent court declared the Will to be valid except that the
devise in favor of the petitioner is null and void pursuant to Article 739 in relation with Article 1028 of
the Civil Code of the Philippines. The dispositive portion of the decision reads:

WHEREFORE, the decision a quo is hereby set aside, the will in question declared
valid except the devise in favor of the appellant which is declared null and void. The
properties so devised are instead passed on in intestacy to the appellant in equal
shares, without pronouncement as to cost.

On June 15, 1982, oppositors Rufina Gomez and her children filed a "Motion for Correction of
Clerical Error" praying that the word "appellant" in the last sentence of the dispositive portion of the
decision be changed to "appellees" so as to read: "The properties so devised are instead passed on
intestacy to the appellees in equal shares, without pronouncement as to costs." The motion was
granted by the respondent court on August 10, 1982.

On August 23, 1982, the petitioner filed a motion for reconsideration. This was denied by the
respondent court in a resolution dated December 28, 1982.
The main issue raised by the petitioner is whether or not the respondent court acted in excess of its
jurisdiction when after declaring the last Will and Testament of the deceased Martin Jugo validly
drawn, it went on to pass upon the intrinsic validity of the testamentary provision in favor of herein
petitioner.

The petitioner submits that the validity of the testamentary provision in her favor cannot be passed
upon and decided in the probate proceedings but in some other proceedings because the only
purpose of the probate of a Will is to establish conclusively as against everyone that a Will was
executed with the formalities required by law and that the testator has the mental capacity to execute
the same. The petitioner further contends that even if the provisions of paragraph 1 of Article 739 of
the Civil Code of the Philippines were applicable, the declaration of its nullity could only be made by
the proper court in a separate action brought by the legal wife for the specific purpose of obtaining a
declaration of the nullity of the testamentary provision in the Will in favor of the person with whom
the testator was allegedly guilty of adultery or concubinage.

The respondents on the other hand contend that the fact that the last Will and Testament itself
expressly admits indubitably on its face the meretricious relationship between the testator and the
petitioner and the fact that petitioner herself initiated the presentation of evidence on her alleged
ignorance of the true civil status of the testator, which led private respondents to present contrary
evidence, merits the application of the doctrine enunciated in Nuguid v. Felix Nuguid, et al. (17
SCRA 449) and Felix Balanay, Jr. v. Hon. Antonio Martinez, et al.(G.R. No. L- 39247, June 27,
1975). Respondents also submit that the admission of the testator of the illicit relationship between
him and the petitioner put in issue the legality of the devise. We agree with the respondents.

The respondent court acted within its jurisdiction when after declaring the Will to be validly drawn, it
went on to pass upon the intrinsic validity of the Will and declared the devise in favor of the petitioner
null and void.

The general rule is that in probate proceedings, the court's area of inquiry is limited to an
examination and resolution of the extrinsic validity of the Will. The rule is expressed thus:

xxx xxx xxx

... It is elementary that a probate decree finally and definitively settles all questions
concerning capacity of the testator and the proper execution and witnessing of his
last Will and testament, irrespective of whether its provisions are valid and
enforceable or otherwise. (Fernandez v. Dimagiba, 21 SCRA 428)

The petition below being for the probate of a Will, the court's area of inquiry is limited
to the extrinsic validity thereof. The testators testamentary capacity and the
compliance with the formal requisites or solemnities prescribed by law are the only
questions presented for the resolution of the court. Any inquiry into
the intrinsic validity or efficacy of the provisions of the will or the legality of any devise
or legacy is premature.

xxx xxx xxx

True or not, the alleged sale is no ground for the dismissal of the petition for probate.
Probate is one thing; the validity of the testamentary provisions is another. The first
decides the execution of the document and the testamentary capacity of the testator;
the second relates to descent and distribution (Sumilang v. Ramagosa, 21 SCRA
1369)
xxx xxx xxx

To establish conclusively as against everyone, and once for all, the facts that a will
was executed with the formalities required by law and that the testator was in a
condition to make a will, is the only purpose of the proceedings under the new code
for the probate of a will. (Sec. 625). The judgment in such proceedings determines
and can determine nothing more. In them the court has no power to pass upon the
validity of any provisions made in the will. It can not decide, for example, that a
certain legacy is void and another one valid. ... (Castaneda v. Alemany, 3 Phil. 426)

The rule, however, is not inflexible and absolute. Given exceptional circumstances, the probate court
is not powerless to do what the situation constrains it to do and pass upon certain provisions of the
Will.

In Nuguid v. Nuguid (17 SCRA 449) cited by the trial court, the testator instituted the petitioner as
universal heir and completely preterited her surviving forced heirs. A will of this nature, no matter
how valid it may appear extrinsically, would be null and void. Separate or latter proceedings to
determine the intrinsic validity of the testamentary provisions would be superfluous.

Even before establishing the formal validity of the will, the Court in Balanay .Jr. v. Martinez (64
SCRA 452) passed upon the validity of its intrinsic provisions.

Invoking "practical considerations", we stated:

The basic issue is whether the probate court erred in passing upon the intrinsic
validity of the will, before ruling on its allowance or formal validity, and in declaring it
void.

We are of the opinion that in view of certain unusual provisions of the will, which are
of dubious legality, and because of the motion to withdraw the petition for probate
(which the lower court assumed to have been filed with the petitioner's authorization)
the trial court acted correctly in passing upon the will's intrinsic validity even before its
formal validity had been established. The probate of a will might become an Idle
ceremony if on its face it appears to be intrinsically void. Where practical
considerations demand that the intrinsic validity of the will be passed upon, even
before it is probated, the court should meet the issue (Nuguid v. Nuguid, 64 O.G.
1527, 17 SCRA 449. Compare with Sumilang vs. Ramagosa L-23135, December 26,
1967, 21 SCRA 1369; Cacho v. Udan L-19996, April 30, 1965, 13 SCRA 693).

There appears to be no more dispute at this time over the extrinsic validity of the Will. Both parties
are agreed that the Will of Martin Jugo was executed with all the formalities required by law and that
the testator had the mental capacity to execute his Will. The petitioner states that she completely
agrees with the respondent court when in resolving the question of whether or not the probate court
correctly denied the probate of Martin Jugo's last Will and Testament, it ruled:

This being so, the will is declared validly drawn. (Page 4, Decision, Annex A of
Petition.)

On the other hand the respondents pray for the affirmance of the Court of Appeals' decision in toto.
The only issue, therefore, is the jurisdiction of the respondent court to declare the testamentary
provision in favor of the petitioner as null and void.

We sustain the respondent court's jurisdiction. As stated in Nuguid v. Nuguid, (supra):

We pause to reflect. If the case were to be remanded for probate of the will, nothing
will be gained. On the contrary, this litigation will be protracted. And for aught that
appears in the record, in the record, in the event of probate or if the court rejects the
will, probability exists that the case will come up once again before us on the same
issue of the intrinsic validity or nullity of the will. Result, waste of time, effort,
expense, plus added anxiety. These are the practical considerations that induce us
to a belief that we might as well meet head-on the issue of the validity of the
provisions of the will in question. (Section 2, Rule 1, Rules of Court. Case, et al. v.
Jugo, et al., 77 Phil. 517, 522). After all, there exists a justiciable controversy crying
for solution.

We see no useful purpose that would be served if we remand the nullified provision to the proper
court in a separate action for that purpose simply because, in the probate of a will, the court does not
ordinarily look into the intrinsic validity of its provisions.

Article 739 of the Civil Code provides:

The following donations shall be void:

(1) Those made between persons who were guilty of adultery or concubinage at the
time of the donation;

(2) Those made between persons found guilty of the same criminal offense, in
consideration thereof;

(3) Those made to a public officer or his wife, descendants and ascendants, by
reason of his office.

In the case referred to in No. 1, the action for declaration of nullity may be brought by
the spouse of the donor or donee; and the guilt of the donor and donee may be
proved by preponderance of evidence in the same action.

Article 1028 of the Civil Code provides:

The prohibitions mentioned in Article 739, concerning donations inter vivos shall
apply to testamentary provisions.

In Article III of the disputed Will, executed on August 15, 1968, or almost six years before the
testator's death on July 16, 1974, Martin Jugo stated that respondent Rufina Gomez was his legal
wife from whom he had been estranged "for so many years." He also declared that respondents
Carmelita Jugo and Oscar Jugo were his legitimate children. In Article IV, he stated that he had been
living as man and wife with the petitioner since 1952. Testator Jugo declared that the petitioner was
entitled to his love and affection. He stated that Nepomuceno represented Jugo as her own husband
but "in truth and in fact, as well as in the eyes of the law, I could not bind her to me in the holy bonds
of matrimony because of my aforementioned previous marriage.
There is no question from the records about the fact of a prior existing marriage when Martin Jugo
executed his Will. There is also no dispute that the petitioner and Mr. Jugo lived together in an
ostensible marital relationship for 22 years until his death.

It is also a fact that on December 2, 1952, Martin Jugo and Sofia J. Nepomuceno contracted a
marriage before the Justice of the Peace of Victoria, Tarlac. The man was then 51 years old while
the woman was 48. Nepomuceno now contends that she acted in good faith for 22 years in the belief
that she was legally married to the testator.

The records do not sustain a finding of innocence or good faith. As argued by the private
respondents:

First. The last will and testament itself expressly admits indubitably on its face the
meretricious relationship between the testator and petitioner, the devisee.

Second. Petitioner herself initiated the presentation of evidence on her alleged


ignorance of the true civil status of the testator, which led private respondents to
present contrary evidence.

In short, the parties themselves dueled on the intrinsic validity of the legacy given in
the will to petitioner by the deceased testator at the start of the proceedings.

Whether or not petitioner knew that testator Martin Jugo, the man he had lived with
as man and wife, as already married, was an important and specific issue brought by
the parties before the trial court, and passed upon by the Court of Appeals.

Instead of limiting herself to proving the extrinsic validity of the will, it was petitioner
who opted to present evidence on her alleged good faith in marrying the testator.
(Testimony of Petitioner, TSN of August 1, 1982, pp. 56-57 and pp. 62-64).

Private respondents, naturally, presented evidence that would refute the testimony of
petitioner on the point.

Sebastian Jugo, younger brother of the deceased testator, testified at length on the
meretricious relationship of his brother and petitioner. (TSN of August 18,1975).

Clearly, the good faith of petitioner was by option of the parties made a decisive
issue right at the inception of the case.

Confronted by the situation, the trial court had to make a ruling on the question.

When the court a quo held that the testator Martin Jugo and petitioner 'were deemed
guilty of adultery or concubinage', it was a finding that petitioner was not the innocent
woman she pretended to be.

xxx xxx xxx

3. If a review of the evidence must be made nonetheless, then private respondents


respectfully offer the following analysis:
FIRST: The secrecy of the marriage of petitioner with the deceased testator in a town
in Tarlac where neither she nor the testator ever resided. If there was nothing to hide
from, why the concealment' ? Of course, it maybe argued that the marriage of the
deceased with private respondent Rufina Gomez was likewise done in secrecy. But it
should be remembered that Rufina Gomez was already in the family way at that time
and it would seem that the parents of Martin Jugo were not in favor of the marriage
so much so that an action in court was brought concerning the marriage. (Testimony
of Sebastian Jugo, TSN of August 18, 1975, pp. 29-30)

SECOND: Petitioner was a sweetheart of the deceased testator when they were still
both single. That would be in 1922 as Martin Jugo married respondent Rufina Gomez
on November 29, 1923 (Exh. 3). Petitioner married the testator only on December 5,
1952. There was a space of about 30 years in between. During those 30 years, could
it be believed that she did not even wonder why Martin Jugo did not marry her nor
contact her anymore after November, 1923 - facts that should impel her to ask her
groom before she married him in secrecy, especially so when she was already about
50 years old at the time of marriage.

THIRD: The fact that petitioner broke off from Martin Jugo in 1923 is by itself
conclusive demonstration that she new that the man she had openly lived for 22
years as man and wife was a married man with already two children.

FOURTH: Having admitted that she knew the children of respondent Rufina Gomez,
is it possible that she would not have asked Martin Jugo whether or not they were his
illegitimate or legitimate children and by whom? That is un-Filipino.

FIFTH: Having often gone to Pasig to the residence of the parents of the deceased
testator, is it possible that she would not have known that the mother of private
respondent Oscar Jugo and Carmelita Jugo was respondent Rufina Gomez,
considering that the houses of the parents of Martin Jugo (where he had lived for
many years) and that of respondent Rufina Gomez were just a few meters away?

Such pretentions of petitioner Sofia Nepomuceno are unbelievable. They are, to say
the least, inherently improbable, for they are against the experience in common life
and the ordinary instincts and promptings of human nature that a woman would not
bother at all to ask the man she was going to marry whether or not he was already
married to another, knowing that her groom had children. It would be a story that
would strain human credulity to the limit if petitioner did not know that Martin Jugo
was already a married man in view of the irrefutable fact that it was precisely his
marriage to respondent Rufina Gomez that led petitioner to break off with the
deceased during their younger years.

Moreover, the prohibition in Article 739 of the Civil Code is against the making of a donation between
persons who are living in adultery or concubinage. It is the donation which becomes void. The giver
cannot give even assuming that the recipient may receive. The very wordings of the Will invalidate
the legacy because the testator admitted he was disposing the properties to a person with whom he
had been living in concubinage.

WHEREFORE, the petition is DISMISSED for lack of merit. The decision of the Court of Appeals,
now Intermediate Appellate Court, is AFFIRMED. No costs.

SO ORDERED.
Teehankee (Chairman), Melencio-Herrera, Plana, Relova, De la Fuente and Patajo, JJ., concur.

G.R. No. L-23445 June 23, 1966

REMEDIOS NUGUID, petitioner and appellant,


vs.
FELIX NUGUID and PAZ SALONGA NUGUID, oppositors and appellees.

Custodio O. Partade for petitioner and appellant.


Beltran, Beltran and Beltran for oppositors and appellees.

SANCHEZ, J.:

Rosario Nuguid, a resident of Quezon City, died on December 30, 1962, single, without
descendants, legitimate or illegitimate. Surviving her were her legitimate parents, Felix Nuguid and
Paz Salonga Nuguid, and six (6) brothers and sisters, namely: Alfredo, Federico, Remedios,
Conrado, Lourdes and Alberto, all surnamed Nuguid.

On May 18, 1963, petitioner Remedios Nuguid filed in the Court of First Instance of Rizal a
holographic will allegedly executed by Rosario Nuguid on November 17, 1951, some 11 years
before her demise. Petitioner prayed that said will be admitted to probate and that letters of
administration with the will annexed be issued to her.

On June 25, 1963, Felix Nuguid and Paz Salonga Nuguid, concededly the legitimate father and
mother of the deceased Rosario Nuguid, entered their opposition to the probate of her will. Ground
therefor, inter alia, is that by the institution of petitioner Remedios Nuguid as universal heir of the
deceased, oppositors who are compulsory heirs of the deceased in the direct ascending line
were illegally preterited and that in consequence the institution is void.

On August 29, 1963, before a hearing was had on the petition for probate and objection thereto,
oppositors moved to dismiss on the ground of absolute preterition.

On September 6, 1963, petitioner registered her opposition to the motion to dismiss. 1wph1.t

The court's order of November 8, 1963, held that "the will in question is a complete nullity and will
perforce create intestacy of the estate of the deceased Rosario Nuguid" and dismissed the petition
without costs.

A motion to reconsider having been thwarted below, petitioner came to this Court on appeal.

1. Right at the outset, a procedural aspect has engaged our attention. The case is for the probate of
a will. The court's area of inquiry is limited to an examination of, and resolution on,
the extrinsic validity of the will. The due execution thereof, the testatrix's testamentary capacity, and
the compliance with the requisites or solemnities by law prescribed, are the questions solely to be
presented, and to be acted upon, by the court. Said court at this stage of the proceedings is not
called upon to rule on the intrinsic validity or efficacy of the provisions of the will, the legality of any
devise or legacy therein.1
A peculiar situation is here thrust upon us. The parties shunted aside the question of whether or not
the will should be allowed probate. For them, the meat of the case is the intrinsic validity of the will.
Normally, this comes only after the court has declared that the will has been duly authenticated.2 But
petitioner and oppositors, in the court below and here on appeal, travelled on the issue of law, to wit:
Is the will intrinsically a nullity?

We pause to reflect. If the case were to be remanded for probate of the will, nothing will be gained.
On the contrary, this litigation will be protracted. And for aught that appears in the record, in the
event of probate or if the court rejects the will, probability exists that the case will come up once
again before us on the same issue of the intrinsic validity or nullity of the will. Result: waste of time,
effort, expense, plus added anxiety. These are the practical considerations that induce us to a belief
that we might as well meet head-on the issue of the validity of the provisions of the will in
question.3 After all, there exists a justiciable controversy crying for solution.

2. Petitioner's sole assignment of error challenges the correctness of the conclusion below that the
will is a complete nullity. This exacts from us a study of the disputed will and the applicable statute.

Reproduced hereunder is the will:

Nov. 17, 1951

I, ROSARIO NUGUID, being of sound and disposing mind and memory, having amassed a certain
amount of property, do hereby give, devise, and bequeath all of the property which I may have when
I die to my beloved sister Remedios Nuguid, age 34, residing with me at 38-B Iriga, Q.C. In witness
whereof, I have signed my name this seventh day of November, nineteen hundred and fifty-one.

(Sgd.) Illegible

T/ ROSARIO NUGUID

The statute we are called upon to apply in Article 854 of the Civil Code which, in part, provides:

ART. 854. The preterition or omission of one, some, or all of the compulsory heirs in the
direct line, whether living at the time of the execution of the will or born after the death of the
testator, shall annul the institution of heir; but the devises and legacies shall be valid insofar
as they are not inofficious. ...

Except for inconsequential variation in terms, the foregoing is a reproduction of Article 814 of the
Civil Code of Spain of 1889, which is similarly herein copied, thus

Art. 814. The preterition of one or all of the forced heirs in the direct line, whether living at the
time of the execution of the will or born after the death of the testator, shall void the institution
of heir; but the legacies and betterments4 shall be valid, in so far as they are not inofficious.
...

A comprehensive understanding of the term preterition employed in the law becomes a necessity.
On this point Manresa comments:

La pretericion consiste en omitar al heredero en el testamento. O no se le nombra siquiera o


aun nombrandole como padre, hijo, etc., no se le instituya heredero ni se le deshereda
expresamente ni se le asigna parte alguna de los bienes, resultando privado de un modo
tacito de su derecho a legitima.

Para que exista pretericion, con arreglo al articulo 814, basta que en el testamento omita el
testador a uno cualquiera de aquellos a quienes por su muerte corresponda la herencia
forzosa.

Se necesita, pues, a) Que la omision se refiera a un heredero forzoso. b) Que la omision sea
completa; que el heredero forzoso nada reciba en el testamento.

It may now appear trite bat nonetheless helpful in giving us a clear perspective of the problem before
us, to have on hand a clear-cut definition of the word annul:

To "annul" means to abrogate, to make void ... In re Morrow's Estate, 54 A. 342, 343, 204
Pa. 484.6

The word "annul" as used in statute requiring court to annul alimony provisions of divorce
decree upon wife's remarriage means to reduce to nothing; to annihilate; obliterate; blot out;
to make void or of no effect; to nullify; to abolish. N.J.S.A. 2:50 38 (now N.J.S. 2A:34-35).
Madden vs. Madden, 40 A. 2d 611, 614, 136 N..J Eq. 132.7

ANNUL. To reduce to nothing; annihilate; obliterate; to make void or of no effect; to nullify; to


abolish; to do away with. Ex parte Mitchell, 123 W. Va. 283, 14 S.E. 2d. 771, 774.8

And now, back to the facts and the law. The deceased Rosario Nuguid left no descendants,
legitimate or illegitimate. But she left forced heirs in the direct ascending line her parents, now
oppositors Felix Nuguid and Paz Salonga Nuguid. And, the will completely omits both of them: They
thus received nothing by the testament; tacitly, they were deprived of their legitime; neither were
they expressly disinherited. This is a clear case of preterition. Such preterition in the words of
Manresa "anulara siempre la institucion de heredero, dando caracter absoluto a este ordenamiento
referring to the mandate of Article 814, now 854 of the Civil Code.9 The one-sentence will here
institutes petitioner as the sole, universal heir nothing more. No specific legacies or bequests are
therein provided for. It is in this posture that we say that the nullity is complete. Perforce, Rosario
Nuguid died intestate. Says Manresa:

En cuanto a la institucion de heredero, se anula. Lo que se anula deja de existir, en todo o


en parte? No se aade limitacion alguna, como en el articulo 851, en el que se expresa que
se anulara la institucion de heredero en cuanto prejudique a la legitima del deseheredado
Debe, pues, entenderse que la anulacion es completa o total, y que este articulo como
especial en el caso que le motiva rige con preferencia al 817. 10

The same view is expressed by Sanchez Roman:

La consecuencia de la anulacion o nulidad de la institucion de heredero por pretericion de


uno, varios o todos los forzosos en linea recta, es la apertura de la sucesion intestada total o
parcial. Sera total, cuando el testador que comete la pretericion, hubiese dispuesto de todos
los bienes por titulo universal de herencia en favor de los herederos instituidos, cuya
institucion se anula, porque asi lo exige la generalidad del precepto legal del art. 814, al
determinar, como efecto de la pretericion, el de que "anulara la institucion de heredero." ... 11
Really, as we analyze the word annul employed in the statute, there is no escaping the conclusion
that the universal institution of petitioner to the entire inheritance results in totally abrogating the will.
Because, the nullification of such institution of universal heir without any other testamentary
disposition in the will amounts to a declaration that nothing at all was written. Carefully worded
and in clear terms, Article 854 offers no leeway for inferential interpretation. Giving it an expansive
meaning will tear up by the roots the fabric of the statute. On this point, Sanchez Roman cites the
"Memoria annual del Tribunal Supreme, correspondiente a 1908", which in our opinion expresses
the rule of interpretation, viz:

... El art. 814, que preceptua en tales casos de pretericion la nulidad de la institucion de
heredero, no consiente interpretacion alguna favorable a la persona instituida en el sentido
antes expuesto aun cuando parezca, y en algun caso pudiera ser, mas o menos equitativa,
porque una nulidad no significa en Derecho sino la suposicion de que el hecho o el acto no
se ha realizado, debiendo por lo tanto procederse sobre tal base o supuesto, y
consiguientemente, en un testamento donde falte la institucion, es obligado llamar a los
herederos forzosos en todo caso, como habria que llamar a los de otra clase, cuando el
testador no hubiese distribudo todos sus bienes en legados, siendo tanto mas obligada esta
consecuencia legal cuanto que, en materia de testamentos, sabido es, segun tiene
declarado la jurisprudencia, con repeticion, que no basta que sea conocida la voluntad de
quien testa si esta voluntad no aparece en la forma y en las condiciones que la ley ha
exigido para que sea valido y eficaz, por lo que constituiria una interpretacion arbitraria,
dentro del derecho positivo, reputar como legatario a un heredero cuya institucion fuese
anulada con pretexto de que esto se acomodaba mejor a la voluntad del testador, pues aun
cuando asi fuese, sera esto razon para modificar la ley, pero no autoriza a una interpretacion
contraria a sus terminos y a los principios que informan la testamentifaccion, pues no porque
parezca mejor una cosa en el terreno del Derecho constituyente, hay razon para convereste
juicio en regla de interpretacion, desvirtuando y anulando por este procedimiento lo que el
legislador quiere establecer. 12

3. We should not be led astray by the statement in Article 854 that, annullment notwithstanding, "the
devises and legacies shall be valid insofar as they are not inofficious". Legacies and devises merit
consideration only when they are so expressly given as such in a will. Nothing in Article 854
suggests that the mere institution of a universal heir in a will void because of preterition would
give the heir so instituted a share in the inheritance. As to him, the will is inexistent. There must be,
in addition to such institution, a testamentary disposition granting him bequests or legacies apart and
separate from the nullified institution of heir. Sanchez Roman, speaking of the two component parts
of Article 814, now 854, states that preterition annuls the institution of the heir "totalmente por la
pretericion"; but added (in reference to legacies and bequests) "pero subsistiendo ... todas aquellas
otras disposiciones que no se refieren a la institucion de heredero ... . 13 As Manresa puts it,
annulment throws open to intestate succession the entire inheritance including "la porcion libre (que)
no hubiese dispuesto en virtud de legado, mejora o donacion. 14

As aforesaid, there is no other provision in the will before us except the institution of petitioner as
universal heir. That institution, by itself, is null and void. And, intestate succession ensues.

4. Petitioner's mainstay is that the present is "a case of ineffective disinheritance rather than one of
preterition". 15From this, petitioner draws the conclusion that Article 854 "does not apply to the case
at bar". This argument fails to appreciate the distinction between pretention and disinheritance.

Preterition "consists in the omission in the testator's will of the forced heirs or anyone of them, either
because they are not mentioned therein, or, though mentioned, they are neither instituted as heirs
nor are expressly disinherited." 16 Disinheritance, in turn, "is a testamentary disposition depriving any
compulsory heir of his share in the legitime for a cause authorized by law. " 17 In Manresa's own
words: "La privacion expresa de la legitima constituye la desheredacion. La privacion tacita de la
misma se denomina pretericion." 18 Sanchez Roman emphasizes the distinction by stating that
disinheritance "es siempre voluntaria"; preterition, upon the other hand, is presumed to be
"involuntaria". 19 Express as disinheritance should be, the same must be supported by a legal cause
specified in the will itself. 20

The will here does not explicitly disinherit the testatrix's parents, the forced heirs. It simply omits their
names altogether. Said will rather than be labeled ineffective disinheritance is clearly one in which
the said forced heirs suffer from preterition.

On top of this is the fact that the effects flowing from preterition are totally different from those of
disinheritance. Preterition under Article 854 of the Civil Code, we repeat, "shall annul the institution
of heir". This annulment is in toto, unless in the will there are, in addition, testamentary dispositions
in the form of devises or legacies. In ineffective disinheritance under Article 918 of the same Code,
such disinheritance shall also "annul the institution of heirs", put only "insofar as it may prejudice the
person disinherited", which last phrase was omitted in the case of preterition. 21 Better stated yet, in
disinheritance the nullity is limited to that portion of the estate of which the disinherited heirs have
been illegally deprived. Manresa's expressive language, in commenting on the rights of the
preterited heirs in the case of preterition on the one hand and legal disinheritance on the other, runs
thus: "Preteridos, adquiren el derecho a todo; desheredados, solo les corresponde un tercio o dos
tercios, 22 el caso.23

5. Petitioner insists that the compulsory heirs ineffectively disinherited are entitled to receive their
legitimes, but that the institution of heir "is not invalidated," although the inheritance of the heir so
instituted is reduced to the extent of said legitimes. 24

This is best answered by a reference to the opinion of Mr. Chief Justice Moran in the Neri case
heretofore cited,viz:

But the theory is advanced that the bequest made by universal title in favor of the children by
the second marriage should be treated as legado and mejora and, accordingly, it must not be
entirely annulled but merely reduced. This theory, if adopted, will result in a complete
abrogation of Articles 814 and 851 of the Civil Code. If every case of institution of heirs may
be made to fall into the concept of legacies and betterments reducing the bequest
accordingly, then the provisions of Articles 814 and 851 regarding total or partial nullity of the
institution, would. be absolutely meaningless and will never have any application at all. And
the remaining provisions contained in said article concerning the reduction of inofficious
legacies or betterments would be a surplusage because they would be absorbed by Article
817. Thus, instead of construing, we would be destroying integral provisions of the Civil
Code.

The destructive effect of the theory thus advanced is due mainly to a failure to distinguish
institution of heirs from legacies and betterments, and a general from a special provision.
With reference to article 814, which is the only provision material to the disposition of this
case, it must be observed that the institution of heirs is therein dealt with as a thing separate
and distinct from legacies or betterments. And they are separate and distinct not only
because they are distinctly and separately treated in said article but because they are in
themselves different. Institution of heirs is a bequest by universal title of property that is
undetermined. Legacy refers to specific property bequeathed by a particular or special title.
... But again an institution of heirs cannot be taken as a legacy. 25
The disputed order, we observe, declares the will in question "a complete nullity". Article 854 of the
Civil Code in turn merely nullifies "the institution of heir". Considering, however, that the will before
us solely provides for the institution of petitioner as universal heir, and nothing more, the result is the
same. The entire will is null.

Upon the view we take of this case, the order of November 8, 1963 under review is hereby affirmed.
No costs allowed. So ordered.

Concepcion, C.J., Reyes, J.B.L., Barrera, Dizon, Regala, Makalintal, Bengzon, J.P. and Zaldivar,
JJ., concur.

G.R. No. L-40207 September 28, 1984

ROSA K. KALAW, petitioner,


vs.
HON. JUDGE BENJAMIN RELOVA, Presiding Judge of the CFI of Batangas, Branch VI, Lipa
City, and GREGORIO K. KALAW, respondents.

Leandro H. Fernandez for petitioner.

Antonio Quintos and Jose M. Yacat for respondents.

MELENCIO-HERRERA, J.:

On September 1, 1971, private respondent GREGORIO K. KALAW, claiming to be the sole heir of
his deceased sister, Natividad K. Kalaw, filed a petition before the Court of First Instance of
Batangas, Branch VI, Lipa City, for the probate of her holographic Will executed on December 24,
1968.

The holographic Will reads in full as follows:

My Last will and Testament

In the name of God, Amen.

I Natividad K. Kalaw Filipino 63years of age, single, and a resident of Lipa City, being of sound and
disposing mind and memory, do hereby declare thus to be my last will and testament.

1. It is my will that I'll be burried in the cemetery of the catholic church of Lipa City. In accordance
with the rights of said Church, and that my executrix hereinafter named provide and erect at the
expose of my state a suitable monument to perpetuate my memory.

xxx xxx xxx

The holographic Will, as first written, named ROSA K. Kalaw, a sister of the testatrix as her sole heir.
Hence, on November 10, 1971, petitioner ROSA K. Kalaw opposed probate alleging, in substance,
that the holographic Will contained alterations, corrections, and insertions without the proper
authentication by the full signature of the testatrix as required by Article 814 of the Civil Code
reading:
Art. 814. In case of any insertion, cancellation, erasure or alteration in a holographic
will the testator must authenticate the same by his full signature.

ROSA's position was that the holographic Will, as first written, should be given effect and probated
so that she could be the sole heir thereunder.

After trial, respondent Judge denied probate in an Order, dated September 3, 197 3, reading in part:

The document Exhibit "C" was submitted to the National Bureau of Investigation for
examination. The NBI reported that the handwriting, the signature, the insertions
and/or additions and the initial were made by one and the same person.
Consequently, Exhibit "C" was the handwriting of the decedent, Natividad K. Kalaw.
The only question is whether the win, Exhibit 'C', should be admitted to probate
although the alterations and/or insertions or additions above-mentioned were not
authenticated by the full signature of the testatrix pursuant to Art. 814 of the Civil
Code. The petitioner contends that the oppositors are estopped to assert the
provision of Art. 814 on the ground that they themselves agreed thru their counsel to
submit the Document to the NBI FOR EXAMINATIONS. This is untenable. The
parties did not agree, nor was it impliedly understood, that the oppositors would be in
estoppel.

The Court finds, therefore, that the provision of Article 814 of the Civil Code is
applicable to Exhibit "C". Finding the insertions, alterations and/or additions in Exhibit
"C" not to be authenticated by the full signature of the testatrix Natividad K. Kalaw,
the Court will deny the admission to probate of Exhibit "C".

WHEREFORE, the petition to probate Exhibit "C" as the holographic will of Natividad
K. Kalaw is hereby denied.

SO ORDERED.

From that Order, GREGORIO moved for reconsideration arguing that since the alterations and/or
insertions were the testatrix, the denial to probate of her holographic Will would be contrary to her
right of testamentary disposition. Reconsideration was denied in an Order, dated November 2, 1973,
on the ground that "Article 814 of the Civil Code being , clear and explicit, (it) requires no necessity
for interpretation."

From that Order, dated September 3, 1973, denying probate, and the Order dated November 2,
1973 denying reconsideration, ROSA filed this Petition for Review on certiorari on the sole legal
question of whether or not theoriginal unaltered text after subsequent alterations and insertions were
voided by the Trial Court for lack of authentication by the full signature of the testatrix, should be
probated or not, with her as sole heir.

Ordinarily, when a number of erasures, corrections, and interlineations made by the testator in a
holographic Will litem not been noted under his signature, ... the Will is not thereby invalidated as a
whole, but at most only as respects the particular words erased, corrected or interlined.1 Manresa gave
an Identical commentary when he said "la omision de la salvedad no anula el testamento, segun la regla de jurisprudencia establecida en la
2
sentencia de 4 de Abril de 1895."

However, when as in this case, the holographic Will in dispute had only one substantial provision,
which was altered by substituting the original heir with another, but which alteration did not carry the
requisite of full authentication by the full signature of the testator, the effect must be that the entire
Will is voided or revoked for the simple reason that nothing remains in the Will after that which could
remain valid. To state that the Will as first written should be given efficacy is to disregard the
seeming change of mind of the testatrix. But that change of mind can neither be given effect
because she failed to authenticate it in the manner required by law by affixing her full signature,

The ruling in Velasco, supra, must be held confined to such insertions, cancellations, erasures or
alterations in a holographic Will, which affect only the efficacy of the altered words themselves but
not the essence and validity of the Will itself. As it is, with the erasures, cancellations and alterations
made by the testatrix herein, her real intention cannot be determined with certitude. As Manresa had
stated in his commentary on Article 688 of the Spanish Civil Code, whence Article 814 of the new
Civil Code was derived:

... No infringe lo dispuesto en este articulo del Codigo (el 688) la sentencia que no
declara la nulidad de un testamento olografo que contenga palabras tachadas,
enmendadas o entre renglones no salvadas por el testador bajo su firnia segun
previene el parrafo tercero del mismo, porque, en realidad, tal omision solo puede
afectar a la validez o eficacia de tales palabras, y nunca al testamento mismo, ya por
estar esa disposicion en parrafo aparte de aquel que determine las condiciones
necesarias para la validez del testamento olografo, ya porque, de admitir lo
contrario, se Ilegaria al absurdo de que pequefias enmiendas no salvadas, que en
nada afectasen a la parte esencial y respectiva del testamento, vinieran a anular
este, y ya porque el precepto contenido en dicho parrafo ha de entenderse en
perfecta armonia y congruencia con el art. 26 de la ley del Notariado que declara
nulas las adiciones apostillas entrerrenglonados, raspaduras y tachados en las
escrituras matrices, siempre que no se salven en la forma prevenida, paro no el
documento que las contenga, y con mayor motivo cuando las palabras enmendadas,
tachadas, o entrerrenglonadas no tengan importancia ni susciten duda alguna
acerca del pensamiento del testador, o constituyan meros accidentes de ortografia o
de purez escrituraria, sin trascendencia alguna(l).

Mas para que sea aplicable la doctrina de excepcion contenida en este ultimo
fallo, es preciso que las tachaduras, enmiendas o entrerrenglonados sin salvar saan
de pala bras que no afecter4 alteren ni uarien de modo substancial la express
voluntad del testador manifiesta en el documento. Asi lo advierte la sentencia de 29
de Noviembre de 1916, que declara nulo un testamento olografo por no estar
salvada por el testador la enmienda del guarismo ultimo del ao en que fue
extendido 3(Emphasis ours).

WHEREFORE, this Petition is hereby dismissed and the Decision of respondent Judge, dated
September 3, 1973, is hereby affirmed in toto. No costs.

SO ORDERED.

Plana, Gutierrez, Jr. and De la Fuente, JJ., concur.

Relova, J., took no part.

G.R. No. L-38338 January 28, 1985

IN THE MATTER OF THE INTESTATE ESTATE OF ANDRES G. DE JESUS AND BIBIANA


ROXAS DE JESUS, SIMEON R. ROXAS & PEDRO ROXAS DE JESUS, petitioners,
vs.
ANDRES R. DE JESUS, JR., respondent.

Raul S. Sison Law Office for petitioners.

Rafael Dinglasan, Jr. for heir M. Roxas.

Ledesma, Guytingco Velasco and Associates for Ledesa and A. R. de Jesus.

GUTIERREZ, JR., J.:

This is a petition for certiorari to set aside the order of respondent Hon. Jose C. Colayco, Presiding
Judge Court of First Instance of Manila, Branch XXI disallowing the probate of the holographic Will of
the deceased Bibiana Roxas de Jesus.

The antecedent facts which led to the filing of this petition are undisputed.

After the death of spouses Andres G. de Jesus and Bibiana Roxas de Jesus, Special Proceeding
No. 81503 entitled "In the Matter of the Intestate Estate of Andres G. de Jesus and Bibiana Roxas
de Jesus" was filed by petitioner Simeon R. Roxas, the brother of the deceased Bibiana Roxas de
Jesus.

On March 26, 1973, petitioner Simeon R. Roxas was appointed administrator. After Letters of
Administration had been granted to the petitioner, he delivered to the lower court a document
purporting to be the holographic Will of the deceased Bibiana Roxas de Jesus. On May 26, 1973,
respondent Judge Jose Colayco set the hearing of the probate of the holographic Win on July 21,
1973.

Petitioner Simeon R. Roxas testified that after his appointment as administrator, he found a
notebook belonging to the deceased Bibiana R. de Jesus and that on pages 21, 22, 23 and 24
thereof, a letter-win addressed to her children and entirely written and signed in the handwriting of
the deceased Bibiana R. de Jesus was found. The will is dated "FEB./61 " and states: "This is my
win which I want to be respected although it is not written by a lawyer. ...

The testimony of Simeon R. Roxas was corroborated by the testimonies of Pedro Roxas de Jesus
and Manuel Roxas de Jesus who likewise testified that the letter dated "FEB./61 " is the holographic
Will of their deceased mother, Bibiana R. de Jesus. Both recognized the handwriting of their mother
and positively Identified her signature. They further testified that their deceased mother understood
English, the language in which the holographic Will is written, and that the date "FEB./61 " was the
date when said Will was executed by their mother.

Respondent Luz R. Henson, another compulsory heir filed an "opposition to probate" assailing the
purported holographic Will of Bibiana R. de Jesus because a it was not executed in accordance with
law, (b) it was executed through force, intimidation and/or under duress, undue influence and
improper pressure, and (c) the alleged testatrix acted by mistake and/or did not intend, nor could
have intended the said Will to be her last Will and testament at the time of its execution.

On August 24, 1973, respondent Judge Jose C. Colayco issued an order allowing the probate of the
holographic Will which he found to have been duly executed in accordance with law.
Respondent Luz Roxas de Jesus filed a motion for reconsideration alleging inter alia that the alleged
holographic Will of the deceased Bibiana R. de Jesus was not dated as required by Article 810 of the
Civil Code. She contends that the law requires that the Will should contain the day, month and year
of its execution and that this should be strictly complied with.

On December 10, 1973, respondent Judge Colayco reconsidered his earlier order and disallowed
the probate of the holographic Will on the ground that the word "dated" has generally been held to
include the month, day, and year. The dispositive portion of the order reads:

WHEREFORE, the document purporting to be the holographic Will of Bibiana Roxas


de Jesus, is hereby disallowed for not having been executed as required by the law.
The order of August 24, 1973 is hereby set aside.

The only issue is whether or not the date "FEB./61 " appearing on the holographic Will of the
deceased Bibiana Roxas de Jesus is a valid compliance with the Article 810 of the Civil Code which
reads:

ART. 810. A person may execute a holographic will which must be entirely written,
dated, and signed by the hand of the testator himself. It is subject to no other form,
and may be made in or out of the Philippines, and need not be witnessed.

The petitioners contend that while Article 685 of the Spanish Civil Code and Article 688 of the Old
Civil Code require the testator to state in his holographic Win the "year, month, and day of its
execution," the present Civil Code omitted the phrase Ao mes y dia and simply requires that the
holographic Will should be dated. The petitioners submit that the liberal construction of the
holographic Will should prevail.

Respondent Luz Henson on the other hand submits that the purported holographic Will is void for
non-compliance with Article 810 of the New Civil Code in that the date must contain the year, month,
and day of its execution. The respondent contends that Article 810 of the Civil Code was patterned
after Section 1277 of the California Code and Section 1588 of the Louisiana Code whose Supreme
Courts had consistently ruled that the required date includes the year, month, and day, and that if
any of these is wanting, the holographic Will is invalid. The respondent further contends that the
petitioner cannot plead liberal construction of Article 810 of the Civil Code because statutes
prescribing the formalities to be observed in the execution of holographic Wills are strictly construed.

We agree with the petitioner.

This will not be the first time that this Court departs from a strict and literal application of the statutory
requirements regarding the due execution of Wills. We should not overlook the liberal trend of the
Civil Code in the manner of execution of Wills, the purpose of which, in case of doubt is to prevent
intestacy

The underlying and fundamental objectives permeating the provisions of the law on
wigs in this Project consists in the liberalization of the manner of their execution with
the end in view of giving the testator more freedom in expressing his last wishes, but
with sufficien safeguards and restrictions to prevent the commission of fraud and the
exercise of undue and improper pressure and influence upon the testator.

This objective is in accord with the modem tendency with respect to the formalities in
the execution of wills. (Report of the Code Commission, p. 103)
In Justice Capistrano's concurring opinion in Heirs of Raymundo Castro v. Bustos (27 SCRA 327) he
emphasized that:

xxx xxx xxx

... The law has a tender regard for the will of the testator expressed in his last will
and testament on the ground that any disposition made by the testator is better than
that which the law can make. For this reason, intestate succession is nothing more
than a disposition based upon the presumed will of the decedent.

Thus, the prevailing policy is to require satisfaction of the legal requirements in order to guard
against fraud and bad faith but without undue or unnecessary curtailment of testamentary
privilege Icasiano v. Icasiano, 11 SCRA 422). If a Will has been executed in substantial compliance
with the formalities of the law, and the possibility of bad faith and fraud in the exercise thereof is
obviated, said Win should be admitted to probate (Rey v. Cartagena 56 Phil. 282). Thus,

xxx xxx xxx

... More than anything else, the facts and circumstances of record are to be
considered in the application of any given rule. If the surrounding circumstances
point to a regular execution of the wilt and the instrument appears to have been
executed substantially in accordance with the requirements of the law, the inclination
should, in the absence of any suggestion of bad faith, forgery or fraud, lean towards
its admission to probate, although the document may suffer from some imperfection
of language, or other non-essential defect. ... (Leynez v. Leynez 68 Phil. 745).

If the testator, in executing his Will, attempts to comply with all the requisites, although compliance is
not literal, it is sufficient if the objective or purpose sought to be accomplished by such requisite is
actually attained by the form followed by the testator.

The purpose of the solemnities surrounding the execution of Wills has been expounded by this Court
in Abangan v. Abanga 40 Phil. 476, where we ruled that:

The object of the solemnities surrounding the execution of wills is to close the door
against bad faith and fraud, to avoid substitution of wills and testaments and to
guaranty their truth and authenticity. ...

In particular, a complete date is required to provide against such contingencies as that of two
competing Wills executed on the same day, or of a testator becoming insane on the day on which a
Will was executed (Velasco v. Lopez, 1 Phil. 720). There is no such contingency in this case.

We have carefully reviewed the records of this case and found no evidence of bad faith and fraud in
its execution nor was there any substitution of Wins and Testaments. There is no question that the
holographic Will of the deceased Bibiana Roxas de Jesus was entirely written, dated, and signed by
the testatrix herself and in a language known to her. There is also no question as to its genuineness
and due execution. All the children of the testatrix agree on the genuineness of the holographic Will
of their mother and that she had the testamentary capacity at the time of the execution of said Will.
The objection interposed by the oppositor-respondent Luz Henson is that the holographic Will is
fatally defective because the date "FEB./61 " appearing on the holographic Will is not sufficient
compliance with Article 810 of the Civil Code. This objection is too technical to be entertained.
As a general rule, the "date" in a holographic Will should include the day, month, and year of its
execution. However, when as in the case at bar, there is no appearance of fraud, bad faith, undue
influence and pressure and the authenticity of the Will is established and the only issue is whether or
not the date "FEB./61" appearing on the holographic Will is a valid compliance with Article 810 of the
Civil Code, probate of the holographic Will should be allowed under the principle of substantial
compliance.

WHEREFORE, the instant petition is GRANTED. The order appealed from is REVERSED and SET
ASIDE and the order allowing the probate of the holographic Will of the deceased Bibiana Roxas de
Jesus is reinstated.

SO ORDERED.

Teehankee (Chairman), Melencio-Herrera, Plana, Relova and De la Fuente, JJ., concur.

SECOND DIVISION

[G.R. No. 129505. January 31, 2000]

OCTAVIO S. MALOLES II, petitioner, vs. PACITA DE LOS REYES PHILLIPS, respondent.

[G.R. No. 133359. January 31, 2000]

OCTAVIO S. MALOLES II, petitioner, vs. COURT OF APPEALS, HON. FERNANDO V. GOROSPE, JR., in his
Official Capacity as Presiding Judge of RTC-Makati, Branch 61, and PACITA PHILLIPS as the alleged
executrix of the alleged will of the late Dr. Arturo de Santos, respondents. Scmis

DECISION

MENDOZA, J.:

These are petitions for review on certiorari of the decisions of the Thirteenth and the Special Eighth
Divisions of the Court of Appeals which ruled that petitioner has no right to intervene in the settlement
of the estate of Dr. Arturo de Santos. The cases were consolidated considering that they involve the
same parties and some of the issues raised are the same.

The facts which gave rise to these two petitions are as follows:

On July 20, 1995, Dr. Arturo de Santos, Filipino and a resident of Makati City, filed a petition for probate
of his will[1] in the Regional Trial Court, Branch 61, Makati, docketed as Sp. Proc. No. M-4223. In his
petition, Dr. De Santos alleged that he had no compulsory heirs; that he had named in his will as sole
legatee and devisee the Arturo de Santos Foundation, Inc.; that he disposed by his will his properties
with an approximate value of not less than P2,000,000.00; and that copies of said will were in the
custody of the named executrix, private respondent Pacita de los Reyes Phillips. A copy of the will[2] was
annexed to the petition for probate.

On February 16, 1996, Judge Fernando V. Gorospe, Jr. of RTC-Makati, Branch 61 issued an order granting
the petition and allowing the will. The order reads:
On 03 August 1995, the Court issued an Order setting the hearing of the petition on 12
September 1995, at 8:30 oclock in the morning, copies of which were served to Arturo
de Santos Foundation, Inc. and Ms. Pacita de los Reyes Phillips (Officers Return, dated
04 September 1995 attached to the records). When the case was called for hearing on
the date set, no oppositor appeared nor any written opposition was ever filed and on
motion of petitioner, he was allowed to adduce his evidence in support of the petition.

Petitioner personally appeared before this Court and was placed on the witness stand
and was directly examined by the Court through "free wheeling" questions and answers
to give this Court a basis to determine the state of mind of the petitioner when he
executed the subject will. After the examination, the Court is convinced that petitioner
is of sound and disposing mind and not acting on duress, menace and undue influence
or fraud, and that petitioner signed his Last Will and Testament on his own free and
voluntary will and that he was neither forced nor influenced by any other person in
signing it. Mis sc

Furthermore, it appears from the petition and the evidence adduced that petitioner in
his lifetime, executed his Last Will and Testament (Exhs. "A", "A-1", "A-2", "A-4", "A-5")
at his residence situated at 9 Bauhinia corner Intsia Streets, Forbes Park, Makati City;
said Last Will and Testament was signed in the presence of his three (3) witnesses,
namely, to wit: Dr. Elpidio Valencia (Exhs. "A-6", "A-7", "A-8", "A-16", "A-16-A"), Atty.
Edward J. Berenguer (Exhs. "A-3", "A-3-A", "A-9", "A-10", & "A-11"), and Atty. Victoria C.
delos Reyes (Exhs. "A-12", "A-13", "A-14", "A-17", & "A-18"), who in turn, in the
presence of the testator and in the presence of each and all of the witnesses signed the
said Last Will and Testament and duly notarized before Notary Public Anna Melissa L.
Rosario (Exh. "A-15"); on the actual execution of the Last Will and Testament, pictures
were taken (Exhs. "B" to "B-3").

Petitioner has no compulsory heirs and Arturo de Santos Foundation, Inc., with address
at No. 9 Bauhinia corner Intsia Streets, Forbes Park, Makati City has been named as sole
legatee and devisee of petitioners properties, real and personal, approximately valued
at not less than P2 million, Ms. Pacita de los Reyes Phillips was designated as executor
and to serve as such without a bond.

From the foregoing facts, the Court finds that the petitioner has substantially
established the material allegations contained in his petition. The Last Will and
Testament having been executed and attested as required by law; that testator at the
time of the execution of the will was of sane mind and/or not mentally incapable to
make a Will; nor was it executed under duress or under the influence of fear or threats;
that it was in writing and executed in the language known and understood by the
testator duly subscribed thereof and attested and subscribed by three (3) credible
witnesses in the presence of the testator and of another; that the testator and all the
attesting witnesses signed the Last Will and Testament freely and voluntarily and that
the testator has intended that the instrument should be his Will at the time of affixing
his signature thereto.
WHEREFORE, as prayed for by the petitioner (testator himself) the petition for the
allowance of the Last Will and Testament of Arturo de Santos is hereby APPROVED and
ALLOWED.

Shortly after the probate of his will, Dr. De Santos died on February 26, 1996.

On April 3, 1996, petitioner Octavio S. Maloles II filed a motion for intervention claiming that, as the only
child of Alicia de Santos (testators sister) and Octavio L. Maloles, Sr., he was the sole full-blooded
nephew and nearest of kin of Dr. De Santos. He likewise alleged that he was a creditor of the testator.
Petitioner thus prayed for the reconsideration of the order allowing the will and the issuance of letters
of administration in his name. Mis spped

On the other hand, private respondent Pacita de los Reyes Phillips, the designated executrix of the will,
filed a motion for the issuance of letters testamentary with Branch 61. Later, however, private
respondent moved to withdraw her motion. This was granted, while petitioner was required to file a
memorandum of authorities in support of his claim that said court (Branch 61) still had jurisdiction to
allow his intervention.[3]

Petitioner filed his memorandum of authorities on May 13, 1996. On the other hand, private
respondent, who earlier withdrew her motion for the issuance of letters testamentary in Branch 61,
refiled a petition for the same purpose with the Regional Trial Court, Makati, which was docketed as Sp.
Proc. No. M-4343 and assigned to Branch 65.

Upon private respondents motion, Judge Salvador Abad Santos of Branch 65 issued an order, dated June
28, 1996, appointing her as special administrator of Dr. De Santoss estate.

On July 29, 1996, petitioner sought to intervene in Sp. Proc. No. M-4343 and to set aside the
appointment of private respondent as special administrator. He reiterated that he was the sole and full
blooded nephew and nearest of kin of the testator; that he came to know of the existence of Sp. Proc.
No. M-4343 only by accident; that the probate proceedings in Sp. Proc. No. M-4223 before Branch 61 of
the same court was still pending; that private respondent misdeclared the true worth of the testators
estate; that private respondent was not fit to be the special administrator of the estate; and that
petitioner should be given letters of administration for the estate of Dr. De Santos.

On August 28, 1996, Judge Abad Santos ordered the transfer of Sp. Proc. No. M-4343 to Branch 61, on
the ground that "[it] is related to the case before Judge Gorospe of RTC Branch 61 . . ."

It appears, however, that in Sp. Proc. No. M-4223, Judge Gorospe had denied on August 26, 1996
petitioners motion for intervention. Petitioner brought this matter to the Court of Appeals which, in a
decision[4] promulgated on February 13, 1998, upheld the denial of petitioners motion for intervention.

Meanwhile, Judge Gorospe issued an order, dated September 4, 1996, returning the records of Sp. Proc.
No. M-4343 to Branch 65 on the ground that there was a pending case involving the Estate of Decedent
Arturo de Santos pending before said court. The order reads: Spped

Acting on the ORDER dated 28 August 1996 of Branch 65, this Court, transferring this
case to this Branch 61 on the ground that this case is related with a case before this
Court, let this case be returned to Branch 65 with the information that there is no
related case involving the ESTATE OF DECEDENT ARTURO DE SANTOS pending before
this Branch.

There is, however, a case filed by ARTURO DE SANTOS, as petitioner under Rule 76 of
the Rules of Court for the Allowance of his will during his lifetime docketed as SP. PROC.
NO. M-4223 which was already decided on 16 February 1996 and has become final.

It is noted on records of Case No. M-4223 that after it became final, herein Petitioner
Pacita de los Reyes Phillips filed a MOTION FOR THE ISSUANCE OF LETTERS
TESTAMENTARY, which was subsequently withdrawn after this Court, during the
hearing, already ruled that the motion could not be admitted as the subject matter
involves a separate case under Rule 78 of the Rules of Court, and movant withdrew her
motion and filed this case (No. 4343).

Octavio de Santos Maloles [II] filed a MOTION FOR INTERVENTION before Case No. M-
4223 and this motion was already DENIED in the order (Branch 61) of 26 August 1996
likewise for the same grounds that the matter is for a separate case to be filed under
Rule 78 of the Rules of Court and cannot be included in this case filed under Rule 76 of
the Rules of Court.

It is further noted that it is a matter of policy that consolidation of cases must be


approved by the Presiding Judges of the affected Branches.

Initially, in his decision dated September 23, 1996,[5] Judge Abad Santos appeared firm in his position
that " . . . it would be improper for (Branch 65) to hear and resolve the petition (Sp. Proc. No. M-4343),"
considering that the probate proceedings were commenced with Branch 61. He thus ordered the
transfer of the records back to the latter branch. However, he later recalled his decision and took
cognizance of the case "to expedite the proceedings." Thus, in his Order, dated October 21, 1996, he
stated:

Considering the refusal of the Hon. Fernando V. Gorospe, Jr. of Branch 61 to continue
hearing this case notwithstanding the fact that said branch began the probate
proceedings of the estate of the deceased and must therefore continue to exercise its
jurisdiction to the exclusion of all others, until the entire estate of the testator had been
partitioned and distributed as per Order dated 23 September 1996, this branch
(Regional Trial Court Branch 65) shall take cognizance of the petition if only to expedite
the proceedings, and under the concept that the Regional Trial Court of Makati City is
but one court. Jo spped

Furnish a copy of this order to the Office of the Chief justice and the Office of the Court
Administrator, of the Supreme Court; the Hon. Fernando V. Gorospe, Jr.; Pacita De Los
Reyes Phillips, Petitioner; and Octavio de Santos Maloles, Intervenor.

On November 4, 1996, Judge Abad Santos granted petitioners motion for intervention. Private
respondent moved for a reconsideration but her motion was denied by the trial court. She then filed a
petition forcertiorari in the Court of Appeals which, on February 26, 1997, rendered a decision[6] setting
aside the trial courts order on the ground that petitioner had not shown any right or interest to
intervene in Sp. Proc. No. M-4343.

Hence, these petitions which raise the following issues:

1. Whether or not the Honorable Regional Trial Court - Makati, Branch 61 has lost
jurisdiction to proceed with the probate proceedings upon its issuance of an order
allowing the will of Dr. Arturo de Santos

2. Whether or not the Honorable (Regional Trial Court - Makati, Branch 65) acquired
jurisdiction over the petition for issuance of letters testamentary filed by (private)
respondent.

3. Whether or not the petitioner, being a creditor of the late Dr. Arturo de Santos, has a
right to intervene and oppose the petition for issuance of letters testamentary filed by
the respondent.

4. Whether or not (private) respondent is guilty of forum shopping in filing her petition
for issuance of letters testamentary with the Regional Trial Court - Makati, Branch 65
knowing fully well that the probate proceedings involving the same testate estate of the
decedent is still pending with the Regional Trial Court - Makati, Branch 61. Spped jo

First. Petitioner contends that the probate proceedings in Branch 61 of RTC-Makati did not terminate
upon the issuance of the order allowing the will of Dr. De Santos. Citing the cases of Santiesteban v.
Santiesteban[7] and Tagle v. Manalo,[8] he argues that the proceedings must continue until the estate is
fully distributed to the lawful heirs, devisees, and legatees of the testator, pursuant to Rule 73, 1 of the
Rules of Court. Consequently, petitioner contends that Branch 65 could not lawfully act upon private
respondents petition for issuance of letters testamentary.

The contention has no merit.

In cases for the probate of wills, it is well-settled that the authority of the court is limited to ascertaining
the extrinsic validity of the will, i.e., whether the testator, being of sound mind, freely executed the will
in accordance with the formalities prescribed by law.[9]

Ordinarily, probate proceedings are instituted only after the death of the testator, so much so that, after
approving and allowing the will, the court proceeds to issue letters testamentary and settle the estate of
the testator. The cases cited by petitioner are of such nature. In fact, in most jurisdictions, courts cannot
entertain a petition for probate of the will of a living testator under the principle of ambulatory nature
of wills.[10]

However, Art. 838 of the Civil Code authorizes the filing of a petition for probate of the will filed by the
testator himself. It provides:

Civil Code, Art. 838. No will shall pass either real or personal property unless it is proved
and allowed in accordance with the Rules of Court.
The testator himself may, during his lifetime, petition the court having jurisdiction for
the allowance of his will. In such case, the pertinent provisions of the Rules of Court for
the allowance of wills after the testators death shall govern. Miso

The Supreme Court shall formulate such additional Rules of Court as may be necessary
for the allowance of wills on petition of the testator.

Subject to the right of appeal, the allowance of the will, either during the lifetime of the
testator or after his death, shall be conclusive as to its due execution.

Rule 76, 1 likewise provides:

Sec. 1 Who may petition for the allowance of will. - Any executor, devisee, or legatee
named in a will, or any other person interested in the estate, may, at any time after the
death of the testator, petition the court having jurisdiction to have the will allowed,
whether the same be in his possession or not, or is lost or destroyed.

The testator himself may, during his lifetime, petition in the court for the allowance of
his will.

The rationale for allowing the probate of wills during the lifetime of testator has been explained by the
Code Commission thus:

Most of the cases that reach the courts involve either the testamentary capacity of the
testator or the formalities adopted in the execution of wills. There are relatively few
cases concerning the intrinsic validity of testamentary dispositions. It is far easier for the
courts to determine the mental condition of a testator during his lifetime than after his
death. Fraud, intimidation and undue influence are minimized. Furthermore, if a will
does not comply with the requirements prescribed by law, the same may be corrected
at once. The probate during the testators life, therefore, will lessen the number of
contest upon wills. Once a will is probated during the lifetime of the testator, the only
questions that may remain for the courts to decide after the testators death will refer to
the intrinsic validity of the testamentary dispositions. It is possible, of course, that even
when the testator himself asks for the allowance of the will, he may be acting under
duress or undue influence, but these are rare cases.

After a will has been probated during the lifetime of the testator, it does not necessarily
mean that he cannot alter or revoke the same before his death. Should he make a new
will, it would also be allowable on his petition, and if he should die before he has had a
chance to present such petition, the ordinary probate proceeding after the testators
death would be in order.[11]

Thus, after the allowance of the will of Dr. De Santos on February 16, 1996, there was nothing else for
Branch 61 to do except to issue a certificate of allowance of the will pursuant to Rule 73, 12 of the Rules
of Court. There is, therefore, no basis for the ruling of Judge Abad Santos of Branch 65 of RTC-Makati
that -Nex old
Branch 61 of the Regional Trial Court of Makati having begun the probate proceedings
of the estate of the deceased, it continues and shall continue to exercise said
jurisdiction to the exclusion of all others. It should be noted that probate proceedings
do not cease upon the allowance or disallowance of a will but continues up to such time
that the entire estate of the testator had been partitioned and distributed.

The fact that the will was allowed during the lifetime of the testator meant merely that
the partition and distribution of the estate was to be suspended until the latters death.
In other words, the petitioner, instead of filing a new petition for the issuance of letters
testamentary, should have simply filed a manifestation for the same purpose in the
probate court.[12]

Petitioner, who defends the order of Branch 65 allowing him to intervene, cites Rule 73, 1 which states:

Where estate of deceased persons settled. - If the decedent is an inhabitant of the


Philippines at the time of his death, whether a citizen or an alien, his will shall be
proved, or letters of administration granted, and his estate settled, in the Court of First
Instance in the province in which he resides at the time of his death, and if he is an
inhabitant of a foreign country, the Court of First Instance of any province in which he
had estate. The court first taking cognizance of the settlement of the estate of a
decedent, shall exercise jurisdiction to the exclusion of all other courts. The jurisdiction
assumed by a court, so far as it depends on the place of residence of the decedent, or of
the location of his estate, shall not be contested in a suit or proceeding, except in an
appeal from that court, in the original case, or when the want of jurisdiction appears on
the record.

The above rule, however, actually provides for the venue of actions for the settlement of the estate of
deceased persons. In Garcia Fule v. Court of Appeals, it was held:[13]

The aforequoted Section 1, Rule 73 (formerly Rule 75, Section 1), specifically the clause
"so far as it depends on the place of residence of the decedent, or of the location of the
state," is in reality a matter of venue, as the caption of the Rule indicates: "Settlement
of Estate of Deceased Persons. Venue and Processes." It could not have been intended
to define the jurisdiction over the subject matter, because such legal provision is
contained in a law of procedure dealing merely with procedural matters. Procedure is
one thing, jurisdiction over the subject matter is another. The power or authority of the
court over the subject matter "existed was fixed before procedure in a given cause
began." That power or authority is not altered or changed by procedure, which simply
directs the manner in which the power or authority shall be fully and justly exercised.
There are cases though that if the power is not exercised conformably with the
provisions of the procedural law, purely, the court attempting to exercise it loses the
power to exercise it legally. However, this does not amount to a loss of jurisdiction over
the subject matter. Rather, it means that the court may thereby lose jurisdiction over
the person or that the judgment may thereby be rendered defective for lack of
something essential to sustain it. The appearance of this provision in the procedural law
at once raises a strong presumption that it has nothing to do with the jurisdiction of the
court over the subject matter. In plain words, it is just a matter of method, of
convenience to the parties. Mani kx
Indeed, the jurisdiction over probate proceedings and settlement of estates with approximate value of
over P100,000.00 (outside Metro Manila) or P200,000.00 (in Metro Manila) belongs to the regional trial
courts under B.P. Blg. 129, as amended. The different branches comprising each court in one judicial
region do not possess jurisdictions independent of and incompatible with each other.[14]

It is noteworthy that, although Rule 73, 1 applies insofar as the venue of the petition for probate of the
will of Dr. De Santos is concerned, it does not bar other branches of the same court from taking
cognizance of the settlement of the estate of the testator after his death. As held in the leading case
of Bacalso v. Ramolote:[15]

The various branches of the Court of First Instance of Cebu under the Fourteenth
Judicial District, are a coordinate and co-equal courts, and the totality of which is only
one Court of First Instance. The jurisdiction is vested in the court, not in the judges. And
when a case is filed in one branch, jurisdiction over the case does not attach to the
branch or judge alone, to the exclusion of the other branches. Trial may be held or
proceedings continue by and before another branch or judge. It is for this reason that
Section 57 of the Judiciary Act expressly grants to the Secretary of Justice, the
administrative right or power to apportion the cases among the different branches, both
for the convenience of the parties and for the coordination of the work by the different
branches of the same court. The apportionment and distribution of cases does not
involve a grant or limitation of jurisdiction, the jurisdiction attaches and continues to be
vested in the Court of First Instance of the province, and the trials may be held by any
branch or judge of the court.

Necessarily, therefore, Branch 65 of the RTC of Makati City has jurisdiction over Sp. Proc. No. M-4343.

Second. Petitioner claims the right to intervene in and oppose the petition for issuance of letters
testamentary filed by private respondent. He argues that, as the nearest next of kin and creditor of the
testator, his interest in the matter is material and direct. In ruling that petitioner has no right to
intervene in the proceedings before Branch 65 of RTC-Makati City, the Court of Appeals held:

The private respondent herein is not an heir or legatee under the will of the decedent
Arturo de Santos. Neither is he a compulsory heir of the latter. As the only and nearest
collateral relative of the decedent, he can inherit from the latter only in case of
intestacy. Since the decedent has left a will which has already been probated and
disposes of all his properties the private respondent can inherit only if the said will is
annulled. His interest in the decedents estate is, therefore, not direct or
immediate. Maniks

His claim to being a creditor of the estate is a belated one, having been raised for the
first time only in his reply to the opposition to his motion to intervene, and, as far as the
records show, not supported by evidence.

. . . . [T]he opposition must come from one with a direct interest in the estate or the will,
and the private respondent has none. Moreover, the ground cited in the private
respondents opposition, that the petitioner has deliberately misdeclared the truth
worth and value of the estate, is not relevant to the question of her competency to act
as executor. Section 2, Rule 76 of the Rules of Court requires only an allegation of the
probable value and character of the property of the estate. The true value can be
determined later on in the course of the settlement of the estate.[16]

Rule 79, 1 provides:

Opposition to issuance of letters testamentary. Simultaneous petition for administration.


- Any person interested in a will may state in writing the grounds why letters
testamentary should not issue to the persons named therein as executors, or any of
them, and the court, after hearing upon notice, shall pass upon the sufficiency of such
grounds. A petition may, at the same time, be filed for letters of administration with the
will annexed.

Under this provision, it has been held that an "interested person" is one who would be benefited by the
estate, such as an heir, or one who has a claim against the estate, such as a creditor, and whose interest
is material and direct, not merely incidental or contingent.[17]

Even if petitioner is the nearest next of kin of Dr. De Santos, he cannot be considered an "heir" of the
testator. It is a fundamental rule of testamentary succession that one who has no compulsory or forced
heirs may dispose of his entire estate by will. Thus, Art. 842 of the Civil Code provides:

One who has no compulsory heirs may dispose by will of all his estate or any part of it in
favor of any person having capacity to succeed. Manikan

One who has compulsory heirs may dispose of his estate provided he does not
contravene the provisions of this Code with regard to the legitimate of said heirs.

Compulsory heirs are limited to the testators -

(1) Legitimate children and descendants, with respect to their legitimate parents and
ascendants;

(2) In default of the foregoing, legitimate parents and ascendants, with respect to their
legitimate children and descendants;

(3) The widow or widower;

(4) Acknowledged natural children, and natural children by legal fiction;

(5) Other illegitimate children referred to in Article 287 of the Civil Code.[18]

Petitioner, as nephew of the testator, is not a compulsory heir who may have been preterited in the
testators will.

Nor does he have any right to intervene in the settlement proceedings based on his allegation that he is
a creditor of the deceased. Since the testator instituted or named an executor in his will, it is incumbent
upon the Court to respect the desires of the testator. As we stated in Ozaeta v. Pecson:[19]
The choice of his executor is a precious prerogative of a testator, a necessary
concomitant of his right to dispose of his property in the manner he wishes. It is natural
that the testator should desire to appoint one of his confidence, one who can be trusted
to carry out his wishes in the disposal of his estate. The curtailment of this right may be
considered a curtailment of the right to dispose.

Only if the appointed executor is incompetent, refuses the trust, or fails to give bond may the court
appoint other persons to administer the estate.[20] None of these circumstances is present in this case.

Third. Petitioner contends that private respondent is guilty of forum shopping when she filed the
petition for issuance of letters testamentary (Sp. Proc. No. M-4343) while the probate proceedings (Sp.
Proc. No. M-4223) were still pending. According to petitioner, there is identity of parties, rights asserted,
and reliefs prayed for in the two actions which are founded on the same facts, and a judgment in either
will result in res judicata in the other.

This contention has no merit. As stated earlier, the petition for probate was filed by Dr. De Santos, the
testator, solely for the purpose of authenticating his will. Upon the allowance of his will, the proceedings
were terminated. Oldmis o

On the other hand, the petition for issuance of letters testamentary was filed by private respondent, as
executor of the estate of Dr. De Santos, for the purpose of securing authority from the Court to
administer the estate and put into effect the will of the testator. The estate settlement proceedings
commenced by the filing of the petition terminates upon the distribution and delivery of the legacies
and devises to the persons named in the will. Clearly, there is no identity between the two petitions, nor
was the latter filed during the pendency of the former. There was, consequently, no forum shopping.

WHEREFORE, the petition is DENIED and the decisions of the Court of Appeals are hereby AFFIRMED.

SO ORDERED.

Bellosillo, (Chairman), Quisumbing, Buena, and De Leon, Jr., JJ., concur.

Das könnte Ihnen auch gefallen